You are on page 1of 56

Mục lục

1 Cực trị trong tổ hợp . . . . . . . . . . . . . . . . . . . . . . . . . . . . . . . . . . 2


I. Bài tập . . . . . . . . . . . . . . . . . . . . . . . . . . . . . . . . . . . . . 18

hu
T
t
Tấ
n
yễ
gu
N

1
1. CỰC TRỊ TRONG TỔ HỢP

§1. Cực trị trong tổ hợp


Bài toán cực trị thường xuyên xuất hiện trong các bài toán tổ hợp. Để giải quyết bài toán cực
trị ta cần vận dụng nhiều nội dung kiến thức liên quan của đại số, tổ hợp. Thông thường ta gặp
các bài toán sau:
Bài toán 1. Tìm số nguyên dương k lớn nhất (nhỏ nhất) sao cho với mọi tập con A của tập X
nào đó mà |A| = k thì A có tính chất T .
Để giải quyết bài toán trên, ta thường đi theo hai hướng sau:
Hướng 1:

• Xây dựng tập A0 ⊂ X mà |A0 | = k0 + 1 và A0 không có tính chất T .

• Chứng minh mọi tập con A của X mà |A| = k0 thì A có tính chất T .

Khi đó kmax = k0 .
Hướng 2:

• Ta chứng minh nếu A có tính chất T thì |A| ≤ k0 .

• Chỉ ra mọi tập A ⊂ X mà |A| = k0 thì A có tính chất T

hu
Khi đó kmax = k0 .

T
Ví dụ 1.1. Gọi A là tập tất cả các số tự nhiên lẻ không chia hết cho 5 và nhỏ hơn 30.
Tìm số k nhỏ nhất sao cho mỗi tập con của A gồm k phần tử đều tồn tại hai số a, b sao
t
.
cho a .. b.
Tấ

Ta có: A = {1, 3, 7, 9, 11, 13, 17, 19, 21, 23, 27, 29} , |A| = 12.
Trước hết ta chỉ ra một tập A0 ⊂ A (có số phần tử càng lớn càng tốt) mà trong A0 không có hai
n

số nào mà số này chia hết cho số kia. Bằng cách thử chọn các giá trị ban đầu, ta thấy tập A0
như sau sẽ có số phần tử nhiều nhất.
yễ

Xét tập A0 = {9, 11, 13, 17, 19, 21, 23, 29}.
Dễ thấy hai phần tử bất kì thuộc A0 thì không chia hết cho nhau. Từ đó ta suy ra được k ≥ 9.
gu

Ta chứng minh k = 9 thỏa đề bài.


Xét S là một tập con bất kì của A và |S| = 9.
N

Xét ba cặp {21, 7} , {27, 9} , {1, 11}, ta thấy mỗi cặp là bội của nhau.
Nếu trong 3 cặp trên có ít nhất một cặp thuộc S thì bài toán được giải quyết.
Giả sử trong ba cặp trên không có cặp nào cùng thuộc S, do |S| = 9 nên S phải chữa một số
trong mỗi cặp và chứa 6 số còn lại. Từ đó suy ra trong S phải có cặp {3; 9} hoặc {3; 27} và
mỗi cặp này là bội của nhau. Hay nói cách khác trong S luôn tồn tại hai số chia hết cho nhau.
Vậy kmin = 9.
Nhận xét. Mẫu chốt trong bài toán trên là chúng ta phát hiện ra tập A0 để từ đó ta khẳng
định được k ≥ 9 và dự đoán kmin = 9. Để tìm tập A0 , ta liệt kê hết các số trong A mà không có
hai số nào là bội của nhau. Với bài toán này, việc tìm ra tập A0 khá đơn giản.

Ví dụ 1.2 (VMO 2004). Cho tập A gồm 16 số nguyên dương đầu tiên. Hãy tìm số
nguyên dương k nhỏ nhất có tính chất: Trong mỗi tập con có k phần tử của A đều tồn tại
hai số phân biệt a, b sao cho a2 + b2 là số nguyên tố.

Giả sử k là số nguyên dương sao cho trong mỗi tập con có k phân tử của tập A đều tồn tại hai
số phân biệt a, b sao cho a2 + b2 là số nguyên tố.
Ta xét tập T gồm các số chẵn thuộc tập A. Khi đó |T | = 8 và với mọi a, b ∈ T ta có a2 + b2 là

2
1. CỰC TRỊ TRONG TỔ HỢP

hợp số, do đó suy ra k ≥ 9.


Xét các cặp số sau:
A = {1; 4} ∪ {3; 2} ∪ {5; 16} ∪ {6; 15} ∪ {7; 12} ∪ {8; 13} ∪ {9; 10} ∪ {11; 14}
Ta thấy tổng bình phương của mỗi cặp trên là một số nguyên tố.
Xét T là một tập con của A và |T | = 9, khi đó theo nguyên lí Dirichlet T sẽ chứa ít nhất một
cặp nói trên, hay nói cách khác trong T luôn tồn tại hai số phân biệt a, b sao cho a2 + b2 là số
nguyên tố.
Vậy kmin = 9.
Nhận xét.
a) Vì giả thiết a2 + b2 là số nguyên tố nên a2 + b2 không thể là số chẵn hay a, b phải khác
tính chẵn, lẻ. Dựa vào đó ta xây dựng được tập T .
b) Để tìm được sự phân hoạch tập A thành hợp của 8 cặp rời nhau như trên ta làm như sau:
Ta liệt kê tất cả các số a1 ∈ A,a2 ∈ A, . . . ,a16 ∈ A sao cho i2 + a2i ( i = 1,16) là số nguyên
tố . Từ đó ta có được sự phân hoạch trên, sự phân hoạch trên không phải là duy nhất.

Ví dụ 1.3 (VMO 2007). Cho một đa giác đều 2007 đỉnh . Tìm số nguyên dương k nhỏ

hu
nhất thảo mãn tính chất: Trong mỗi cách chọn k đỉnh của đa giác, luôn tồn tại 4 đỉnh tạo
thành một tứ giác lồi mà 3 trong số 4 cạnh của tứ giác là cạnh của đa giác đã cho.

T
Gọi các đỉnh của đa giác là A1 ,A2 , . . . ,A2007 . Ta thấy tứ giác có 4 đỉnh thuộc các đỉnh của đa
giác có 3 cạnh trong 4 cạnh là cạnh của đa giác khi và chỉ khi 4 đỉnh của tứ giác là 4 đỉnh liên
t
tiếp của đa giác.
Tấ

Xét tập hợp X = {A1 ,A2 ,A3 ,A4 , . . . ,A2005 ,A2006 }( bỏ đi các đỉnh A4i và A2007 ). Ta có |X| = 1505
và X không chứa 4 đỉnh liên tiếp của đa giác. Từ đó suy ra k ≥ 1506. Ta chứng minh kmin = 1506.
Gọi T là tập hợp các điểm thuộc đỉnh của đa giác và |T | = 1506. Ta xét 2007 − 1506 = 501 đỉnh
n

 sẽchia đường tròn ngoại tiếp đa giác thành 501 cung, do đó sẽ có một
còn lại. Các đỉnh này
2007
yễ

cung chứa ít nhất + 1 = 4 đỉnh liên tiếp của đa giác. Dĩ nhiên 4 đỉnh này thuộc T và là
501
4 đỉnh liên tiếp của đa giác đã cho.
gu

Vậy kmin = 1506.


Nhận xét.
N

a) Để chứng minh kmin = 1506 ta có thể làm theo cách sau:


Đặt T1 = {A1 ,A2 ,A3 ,A4 } , T2 = {A5 ,A6 ,A7 ,A8 } ,... T501 = {A2001 ,A2002 ,A2003 ,A2004 } ,T502 =
{A2005 ,A2006 ,A2007 }.
Nếu có Ti ⊂ T,i = 1,501 thì bài toán được chứng minh.
Giả sử Ti 6⊂ T,i = 1,501, vì |T | = 1006 nên |T ∩ Ti | = 3, ∀i = 1,501 và T502 ⊂ T .
Vì A2005 ,A2006 ,A2007 ∈ T nên A1 ∈ / T ⇒ A2 ,A3 ,A4 ∈ T ⇒ A5 ∈ / T... ta suy ra được
A2001 ∈
/ T ⇒ A2002 ,A2003 ,A2004 ∈ T .
Do đó A2002 ,A2003 ,A2004 ,A2005 ∈ T . Bài toán được chứng minh.
b) Mẫu chốt bài toán trên là chúng ta đưa ra được nhận xét: Đa giác thỏa yêu cầu bài toán
khi và chỉ khi 4 đỉnh của tứ giác là 4 đỉnh liên tiếp của đa giác . Từ đó chúng ta xây dựng
tập X không thỏa yêu cầu bài toán. Khi xây dựng tập X ta chú ý, cần xây dựng X sao
cho trong X không chứa 4 đỉnh liên tiếp và X có số phần tử lớn nhất.

Ví dụ 1.4. Cho m là một số nguyên dương m ≥ 3. Một tập S được gọi là “tốt” nếu có m
phần tử s1 , s2 , · · · , sm thuộc tập S (không nhất thiết phân biệt) sao cho s1 +s2 +· · ·+sm−1 =
sm . Tìm giá trị nhỏ nhất của f (m) sao cho mọi cách phân hoạch tập X = {1; 2; . . . ; f (m)}
thành hai tập A, B luôn tồn tại ít nhất một trong hai tập A hoặc B là tập “tốt”.

3
1. CỰC TRỊ TRONG TỔ HỢP

Với f (m) = m2 − m − 2, ta xét tập X = {1; 2, · · · ; m2 − m − 2}. Ta phân hoạch tập X


thành hai tập A, B với

A = 1; 2, . . . ; m − 2; (m − 1)2 ; (m − 1)2 + 1; . . . ,m2 − m − 2





B = m − 1; m; . . . ; m2 − 2m .


Xét m − 1 phần tử a1 , a2 , . . . , am ∈ A.
Nếu a1 ,a2 , . . . ,am−1 thuộc tập {1; 2; . . . ; m − 2} thì a1 + a2 + · · · + am−1 có giá trị nằm trong
đoạn từ m − 1 đến (m − 1) (m − 2) nên

a1 + a2 + · · · + am−1 6= am .

Nếu ∃ai ∈
/ {1; 2; · · · ; m − 2} với 1 ≤ i ≤ m − 1 thì

a1 + a2 + · · · + am−1 ≥ (m − 1)2 + m − 2 > m2 − m − 2.

Do đó a1 + a2 + · · · + am−1 6= am . Vậy A không là tập tốt.


Tương tự, với m − 1 phân tử thuộc B thì tổng của chúng không nhỏ hơn

hu
(m − 1) (m − 1) = (m − 1)2 > m2 − 2m

Do đó B không là tập “tốt”.


Suy ra f (m) ≤ m2 − m − 2 không thỏa bài toán.
T
Do đó f (m) ≥ m2 − m − 1. Xét X = {1; 2; . . . ; m2 − m − 1} = A ∪ B.
t
Ta chứng minh trong hai tập A và B có ít nhất một tập “tốt”.
Tấ

Giả sử A và B đều không là tập “tốt”.


Không mất tính tổng quát, ta giả sử 1 ∈ A , suy ra m − 1 ∈ B và (m − 1)2 ∈ A.
Ta lại có
m2 − m − 1 = (m − 1)2 + (m − 2) = (m − 1)2 + 1 + · · · + 1 ∈ B
n

và m + m + · · · + m + 1 = m(m − 2) + 1 = (m − 1)2 nên m ∈ B.


yễ

Khi đó m + m + · · · + m + (m − 1) = m (m − 2) + m − 1 = m2 − m − 1 là tổng của m − 1 phần


tử thuộc B.
gu

Suy ra B là tập “tốt”.


N

Ví dụ 1.5 (IMO Sortlist 2011). Tìm số nguyên dương k lớn nhất sao cho có thể phân
hoạch tập các số tự nhiên thành k tập A1 ,A2 , . . . ,Ak sao cho với mọi số tự nhiên n ≥ 15
và mọi i ∈ {1,2, . . . ,k} luôn tồn tại hai phần tử phân biệt của tập Ai có tổng bằng n.

Xét 3 tập
A1 = {1,2,3} ∪ {3m : m ≥ 4}
A2 = {4,5,6} ∪ {3m − 1 : m ≥ 4}
A2 = {7,8,9} ∪ {3m − 2 : m ≥ 4} .
Khi đó, tập A1 gồm các số tự nhiên n ≥ 13, tập A2 gồm các số tự nhiên n ≥ 15, tập A3 gồm các
số tự nhiên n ≥ 15.
Do đó, k = 3 thỏa yêu cầu bài toán.
Xét k ≥ 4, giả sử tồn tại k tập A1 ,A2 , . . . ,Ak thỏa bài toán. Khi đó, các tập A1 ,A2 ,A3 ,A4 ∪. . .∪Ak
cũng thỏa yêu cầu bài toán nên ta chỉ cần xét k = 4. Đặt Bi = Ai ∩ X, X = {1,2, . . . ,23} với
i = 1,2,3,4.
Vì mỗi số 15,16, . . . ,24 là tổng của của 2 số thuộc tập Bi . Do đó, mỗi tập Bi có ít nhất 5 phần
tử.

|B1 | + |B2 | + |B3 | + |B4 | = 23

4
1. CỰC TRỊ TRONG TỔ HỢP

nên tồn tại chỉ số j sao cho |Bj | = 5.


Đặt Bj = {x1 ,x2 ,x3 ,x4 ,x5 }.
Vì tổng của hai số bất kì khác nhau và thuộc vào tập {15,16, . . . ,24} nên ta có

4 (x1 + x2 + x3 + x4 + x5 ) = 15 + 16 + . . . + 24 = 195.

Vì 195 không chia hết cho 4 nên ta suy ra điều vô lí. Vậy k = 3 là giá trị cần tìm.

Ví dụ 1.6. Cho tập hợp X = {1; 2; 3; . . . ; 2018} gồm 2018 số nguyên dương đầu tiên.
A là một tập con của tập X thỏa mãn: với x, y, z ∈ A; x < y < z thì x, y, z là độ dài ba
cạnh của một tam giác. Hỏi tập hợp A có nhiều nhất bao nhiêu phần tử?

Giả sử A = {a1 ; a2 ; . . . ; ak } với a1 < a2 < · · · < ak . Nếu ak < 2018, đặt t = 2018 − ak và
bi = ai + t với i = 1, 2, . . . , k và tập {b1 ; b2 ; . . . ; bk } cũng thỏa mãn bài toán và bk = 2018. Do
đó ta có thể giả sử ak = 2018.
Ta có a1 + a2 > ak và a1 < a2 nên

2a2 > ak = 2018 ⇒ a2 > 1009 ⇒ a2 ≥ 1010.

hu
Từ đây ta có A\ {a1 } ⊂ {1010; 1011; . . . ; 2018}, do đó

|A| − 1 ≤ 2018 − 1010 + 1 = 1009 ⇒ |A| ≤ 1010.

T
Xét tập hợp A0 = {1009; 1010; . . . ; 2018}, ta có |a0 | = 1010 và với x, y, z ∈ A0 mà x < y < z
thì ta có
t
x + y ≥ 1009 + 1010 = 2019 > 2018 ≥ z,
Tấ

hay x, y, z là độ dài ba cạnh của một tam giác.


Vậy max |a| = 1010.
n

Ví dụ 1.7. Cho bảng (n2 + n + 1) × (n2 + n + 1). Ta điền vào mỗi số của bảng số 0 hoặc
yễ

số 1, sao cho không có bốn ô ghi số 1 nào là đỉnh của hình chữ nhật. Hỏi có nhiều nhất
bao nhiêu chữ số 1 được ghi lên bảng.
gu

Gọi xi là số số 1 ở hàng thứ i. Ta cần tìm max S với


N

n2X
+n+1
S= xi .
i=1

Gọi tập M gồm các cặp (k; l) mà 1 ≤ k < l ≤ n2 + n + 1, ta có

|M | = Cn22 +n+1 .

Với mỗi i = 1,2, . . . ,n2 + n + 1 ta xét tập Mi gồm các cặp (k; l) mà hai ô giao của cột k và cột l
với hàng thứ i đều chứa số 1. Ta có

xi (xi − 1)
|Mi | = Cx2i = .
2
Vì không có 4 ô ghi số 1 là đỉnh của hình chữ nhật nên Mi ∩ Mj = ∅ ∀i 6= j.
Do đó
n2X
+n+1
|Mi | ≤ |M | .
i=1

5
1. CỰC TRỊ TRONG TỔ HỢP

Hay
n
1X 2  (n2 + n) (n2 + n + 1)
xi − xi ≤
2 i=1 2
n2X
+n+1 n2X
+n+1
x2i xi ≤ n2 + n n2 + n + 1 .
 
⇔ −
i=1 i=1

Mặt khác
n2X
+n+1 n2X
+n+1
!2
1
x2i ≥ 2 xi
i=1
n +n+1 i=1

nên ta có
1 2 2
 2 
S − S ≤ n + n n + n + 1
n2 + n + 1
2
⇔ S 2 − n2 + n + 1 S − n2 + n n2 + n + 1 ≤ 0
 

⇒ S ≤ (n + 1) n2 + n + 1 .


Đẳng thức xảy ra khi x1 = x2 = · · · = xn2 +n+1 = n + 1.


Vậy có nhiều nhất (n + 1) (n2 + n + 1) số 1 được ghi lên bảng.

hu
Ví dụ 1.8. Cho các số nguyên 1 ≤ a1 < a2 < · · · < an = 100 thỏa mãn: Với mọi i ≥ 2,
luôn tồn tại 1 ≤ p ≤ q ≤ r ≤ i − 1 sao cho

T
ai = ap + aq + ar .
t
Tấ

Tìm giá trị nhỏ nhất và giá trị lớn nhất của n?

Ta thấy n 6= 1, 2 nên n ≥ 3. Xét ba số a1 = 20, a2 = 60, a3 = 100, ta có a2 = a1 + a1 + a1 và


a3 = a2 + a1 + a1 .
n

Do đó min n = 3.
yễ

Ta thấy nếu a1 ≡ 1 (mod 2) thì a2 = 3a1 ≡ 1 (mod 2).


Bằng quy nạp ta có ai ≡ 1 (mod 2) ∀i = 1,2, . . . , n.
gu

Dẫn đến an = 100 ≡ 1 (mod 2) (vô lí). Do đó a1 ≡ 0 (mod 2) ⇒ a1 ≡ 0, 2 (mod 4).


Nếu a1 ≡ 2 (mod 4) thì chứng minh tương tự ta có an = 100 ≡ 2 (mod 4) (vô lí). Suy ra a1 ≡ 0
(mod 4) hay a1 ≡ 0, 4 (mod 8).
N

Chứng minh tương tự ta suy ra a1 ≡ 4 (mod 8) và ai ≡ 4 (mod 8) ∀i = 1,2, . . . ,n.


Suy ra
.
ai+1 − ai .. 8 ⇒ ai+1 − ai ≥ 8 ⇒ ai+1 ≥ ai + 8.
Do đó
100 = an ≥ a1 + 8(n − 1) ≥ 4 + 8(n − 1) = 8n − 4 ⇒ n ≤ 13.
Xét các số ai = 8i − 4 ∀i = 1,2,...,13 ta có
ai+1 = ai + 8 = ai + a1 + a1 .
Vậy max n = 13.

Ví dụ 1.9 (IMO Shortlist 2012 C2). Cho n ≥ 1 là một số nguyên dương. Xét tập
X = {1, 2, 3, . . . , n} và gọi A là tập các cặp phân biệt thuộc X sao cho tổng của các cặp
khác nhau và không vượt quá n. Tìm giá trị lớn nhất của |A|?

Đặt |A| = x và S là tổng của các phần tử trong tập A, khi đó


2x (2x + 1)
S ≥ 1 + 2 + · · · + 2x = = x (2x + 1) .
2
6
1. CỰC TRỊ TRONG TỔ HỢP

Mặt khác
x (x − 1)
S ≤ (n + n − 1) + (n − 2 + n − 3) + · · · + n − x + 1 = nx − .
2
Suy ra
x (x − 1) 2n − 1
x (2x + 1) ≤ nx − ⇔x≤ .
2 5
Vì x là số nguyên dương nên  
2n − 1
x≤ .
5
 
2n − 1
Ta chứng minh xmax = .
5  
2n − 1
Trước hết ta xét n = 5k + 3, khi đó = 2k + 1. Xét có các cặp sau
5

3k + 1 3k ··· 2k + 2 4k + 2 4k + 1 ··· 3k + 3 3k + 2
2 4 ··· 2k 1 3 ··· 2k − 1 2k + 1
3k + 3 3k + 4 ··· 4k + 2 4k + 3 4k + 4 ··· 5k + 2 5k + 3

hu
Các cặp này gồm các số từ 1 cho đến 4k + 2 và tổng của mỗi cặp nhận giá trị từ 3k + 3 đến
5k + 3.
Với trường hợp n = 5k + 4,n = 5k + 5 thì

2n − 1
5

T
= 2k + 1. Do đó, các chia các cặp như trên
t
thỏa bài toán.
Tấ
 
2n − 1
Với n = 5k + 2 ⇒ = 2k nên trong cách chia trong trường hợp n = 5k + 3 ta xóa đi cặp
5
(3k + 2,2k + 1) ta được 2k cặp thỏa yêu cầu bài toán.
n

Với n = 5k + 1 thì ta xóa cặp (3k + 2,2k + 1) và hàng đầu tiên trong bảng ta trừ đi 1. Khi đó ta
thu được 2k cặp thỏa bài toán.
yễ

2n − 1
Vậy xmax = .
5
gu
N

7
1. CỰC TRỊ TRONG TỔ HỢP

Ví dụ 1.10 (VMO 2017). Cho số nguyên n > 1. Bảng vuông ABCD kích thước n × n
gồm n2 ô vuông đơn vị, mỗi ô vuông đơn vị được tô bởi một trong ba màu: đen, trắng,
xám. Một cách tô màu được gọi là đối xứng nếu mỗi ô có tâm trên đường chéo AC được
tô màu xám và mỗi cặp ô đối xứng qua AC được tô cùng màu đen hoặc cùng màu trắng.
Người ta điền vào mỗi ô xám số 0, mỗi ô trằng một số nguyên dương và mỗi ô đen mộ số
nguyên âm. Một cách điền số như vậy được gọi là k− cân đối (với k nguyên dương) nếu
thỏa mãn các điều kiện sau:

i) Mỗi cặp ô đối xứng qua AC được điền cùng một số nguyên thuộc đoạn [−k; k].

ii) Nếu một hàng và một cột giao nhau tại ô đen thì tập các số nguyên dương được điền
trên hàng đó và tập các số nguyên dương được điền trên cột đó không giao nhau; nếu
một hàng và một cột giao nhau tại ô trắng thì tập các số nguyên âm được điền trên
hàng đó và tập các số nguyên âm được điền trên cột đó không giao nhau.

a) Với n = 5, tìm giá trị nhỏ nhất của k để tồn tại cách điền số k− cân đối cho các tô
màu đối xứng ở hình bên.

hu
T
t
Tấ
n
yễ

b) Với n = 2017, tìm giá trị nhỏ nhất của k để với mọi các tô màu đối xứng, luôn tồn
gu

tại các điền số k− cân đối.


N

a) Gọi Ai là tập các số nguyên dương được ghi trên hàng i với i = 1, 2, 3, 4, 5.
Theo hình như đề bài ta có A1 ∩ A3 = ∅ vì hàng 1, cột 3 giao nhau tại ô đen và cột 3 thì
giống như hàng 3. Tương tự A1 ∩ A4 = A3 ∩ A5 = ∅. Ngoài ra, hàng 1, cột 5 có chung ô
trắng ở góc nên hàng 1, hàng 5 thỏa mãn A1 ∩A5 6= ∅. Tương tự A3 ∩A4 6= ∅, A4 ∩A5 = ∅.
Trước hết, k = 1 không đủ vì không thể điền cùng số 1 để cho A1 ∩ A3 = ∅.
Giả sử với k = 2 điền được, khi đó |A1 | = 1, vì nếu |A1 | = 2 thì không còn số để chọn A3 .
Giả sử A1 = {1}, thế thì A3 = A4 = {2} và A5 = {1}. Nhưng A4 ∩ A5 6= ∅ nên dẫn đến
mẫu thuẫn.
Xét k = 3. Ta xét cách điền sau

0 1 −1 −2 1
1 0 2 2 3
−1 2 0 2 −3
−2 2 2 0 3
1 3 −3 3 0

Vậy kmin = 3.

8
1. CỰC TRỊ TRONG TỔ HỢP

b) Trước hết ta xét cách tô màu như bàn cờ, tức là các ô trắng đen xen kẽ nhau như hình vẽ,
trong đó ô (i,j) được tô màu đen nếu i + j là số chẵn và tô màu trắng nếu i + j là số lẻ.

hu
Xét hai ô trắng bất kỳ trong bảng ô vuông trên ở vị trí (a; b) và (c; d) với 1 ≤ a, b, c, d ≤
2017.

T
• Nếu a + c chẵn thì b + d cũng chẵn, suy ra a + d và b + c lẻ. Khi đó, một trong hai
ô (a; d) và (b; c) sẽ được tô đen vì chúng không thể cùng nằm trên đường chéo màu
t
xám. Suy ra hai ô vuông trắng phải được điền số khác nhau.
Tấ

• Nếu a + c lẻ thì b + d cũng lẻ, xét ô (d; c) điền cùng số với ô (c; d) thì rõ ràng ta có
thể áp dụng lập luận trên để suy ra hai số điền cho hai ô hai khác nhau.
n

Từ đó suy ra tất cả các số điền cho các ô trắng nằm ở nửa trên bên phải của bảng là đôi
yễ

một phân biệt. Do đó, ta thu được kết quả

20172 − 1
gu

k ≥ 2 + 4 + 6 + · · · + 2016 = 1008 · 1009 = .


4
20172 − 1
N

Ta chứng minh k = thỏa mãn bài toán bằng quy nạp rằng kết quả trên cũng
4  2
n
đúng với mọi bảng có kích thước n × n với n là số nguyên dương, cụ thể là k = .
4
Thật vậy, với n = 1, n = 2, n = 3 ta dễ dàng kiểm tra được các kết quả tương ứng.
Xét n ≥ 4 và giả sử khẳng định đúng với mọi số nguyên dương bé hơn n. Đánh số các hàng
và cột từ 1 → n. Ta sẽ chứng minh rằng với mọi vị trí của các ô đen thì luôn tồn tại cách
điền các số nguyên dương không vượt quá kn vào ô trắng còn lại trong bảng (trường hợp
điền số âm thì tương tự vì tính bình đẳng).
Xét graph G = (V, E) mà V là tập hợp các đỉnh, đỉnh thứ i ứng với hàng i và 1 ≤ i ≤ n;
còn E là tập hợp các cạnh, trong đó có cạnh nối từ đỉnh thứ i đến đỉnh thứ j nếu như tại
ô (i; j) và ô (j; i) là ô màu trắng.
Ta xét các trường hợp sau:

• Nếu graph
 G không có chứa tam giác, theo
 2 định lí Mantel-Turan thì nó sẽ có
 không
n2 n2

n
quá cạnh, nghĩa là có không quá ô trắng, nên có thể dùng k = số
4 4 4
nguyên dương điền vào các ô đó.

9
1. CỰC TRỊ TRONG TỔ HỢP

• Nếu graph G có chứa tam giác, giả sử các đỉnh a; b; c phân biệt được nối với nhau đôi
một. Điều này tương ứng với việc các ô (a; b), (b; c), (c; a) và (b; a), (c; b), (a; c) là
giao điểm của các hàng a; b; c đều được tô màu trắng. Khi đó, các số điền vào các ô
đó không cần phải phân biệt và tập hợp các ô trắng (nếu có) còn lại trên các hàng a;
b; c cũng không cần phải rời nhau. Rõ ràng trên mỗi hàng sẽ còn lại không quá n − 3
ô như thế. Khi đó, ta có thể dùng 1 số để điền vào các ô trắng ở trên và dùng không
quá n − 3 số phân biệt để điền vào mỗi ô còn lại của mỗi hàng.
 a; b; c, ta còn lại n − 3 hàng, sử dụng giả thiết quy nạp thì
Nếu không tính 3 hàng
2
(n − 3)
cần không số nguyên dương phân biệt cho các hàng đó.
4
Do đó, trong trường hợp này, số lượng số nguyên dương phân biệt cần dùng không
vượt quá

(n − 3)2 (n − 3)2 + 4(n − 2) (n − 1)2


       2
n
1+n−3+ = = ≤ .
4 4 4 4

n2
 
Tóm lại, trong mọi trường hợp, ta đều cần sử dụng không quá số nguyên dương phân
 2 4
n

hu
biệt. để điền vào các ô trắng hay nói cách khác k = cũng thỏa mãn đề bài với bảng
4
n × n.
20172 − 1
Vậy k =
4
là giá trị cần tìm.
T
t
Tấ

Ví dụ 1.11 (P20, Tạp chí Pi, tháng 1 năm 2017). Trong mặt phẳng, xét 7 điểm đôi
một phân biệt. Người ta muốn vẽ các đường tròn đi qua đúng 4 trong 7 điểm này. Hỏi có
thể vẽ được nhiều nhất bao nhiêu đường tròn?
n

Kí hiệu 7 điểm đang xét là Ai , i = 1,7.


yễ

Giả sử vẽ được n đường tròn, n ∈ N∗ , thỏa mãn điều kiện đề bài.


Với mỗi i = 1,7, kí hiệu si là số đường tròn qua điểm Ai .
gu

7
P
Do mỗi đường tròn đi qua đúng 4 điểm nên si = 4n. Với mỗi i = 1,7, gọi mi là số cặp đường
i=1
7
N

P
tròn mà hai đường tròn trong mỗi cặp cùng đi qua điểm Ai . Đặt m = mi , ta có
i=1

7 7 7
!
X
2 1 X X
m= C si = si 2 − si
i=1
2 i=2 i=1
7
1X 2
= si − 2n
2 i=1
1 (4n)2 2n(4n − 7)
≥ · − 2n = . (1)
2 7 7
7
 7 2
P 2 1 P
(do si ≥ · si ).
i=1 7 i=1
Mặt khác, do 2 đường tròn tùy ý (trong n đường tròn vẽ được) cắt nhau ở tối đa 2 điểm trong
số 7 điểm Ai nên
m ≤ 2Cn2 = n(n − 1). (2)

2n (4n − 7)
Từ (1) và (2), suy ra ≤ n(n − 1). (3)
7
10
1. CỰC TRỊ TRONG TỔ HỢP

Ta có: (3) ⇔ n ≤ 7.
Xét n = 7. Khi đó, ở tất cả các đánh giá nêu trên phải xảy ra dấu “=”. Vì thế, 7 đường tròn vẽ
được phải thỏa mãn đồng thời các điều kiện sau:

1/ Mỗi đường tròn đi qua đúng 4 điểm và qua mỗi điểm có đúng 4 đường tròn;

2/ Hai đường tròn tùy ý cắt nhau ở đúng 2 điểm trong 7 điểm đang xét.

Xét một điểm tùy ý trong 7 điểm, giả sử là A1 . Vì 2/ nên phép nghịch đảo P tâm A1 , tỉ số k 6= 0,
sẽ biến 4 đường tròn đi qua A1 thành 4 đường thẳng không đi qua A1 và đôi một cắt nhau, tạo
thành một tứ giác toàn phần như Hình 1 dưới đây.
Vì mỗi đường tròn trong 3 đường tròn không đi qua A1 , đều đi qua đúng 4 điểm trong số 6 điểm
khác A1 (theo 1/) và đều cắt từng đường tròn trong 6 đường tròn còn lại tại 2 điểm trong 6
điểm đó (theo 2/) nên phép nghịch đảo P biến 3 đường tròn không đi qua A1 thành 3 đường
tròn phân biệt, mỗi đường tròn đi qua đúng 4 điểm trong 6 điểm A, B, C, D, E, F ở hình trên.
Tuy nhiên, dễ thấy, chỉ có thể có tối đa 2 đường tròn phân biệt, mỗi đường tròn đi qua 4 điểm
trong 6 điểm vừa nêu. Mâu thuẫn nhận được chứng tỏ n 6= 7. Do đó, n ≤ 6.

hu
A1

A T
t
B
Tấ

D
C
n

F
yễ

Hình 1
gu

Xét 7 điểm gồm 3 đỉnh của tam giác nhọn ABC, trực tâm H của tam giác đó và 3 chân đường
cao D, E, F hạ từ A, B, C xuống các cạnh BC, CA, AB (xem Hình 2 dưới đây).
N

E
F
H

B D C

11
1. CỰC TRỊ TRONG TỔ HỢP

Hình 2
Dễ thấy, 6 đường tròn (AF HE), (BF HD), (CDHE), (ABDE), (BF EC) và (AF DC) thỏa
mãn các điều kiện của đề bài. Vậy, chỉ có thể vẽ được nhiều nhất 6 đường tròn thỏa mãn các yêu
cầu của đề bài.
Nhận xét. Mặt dù thời hạn gửi lời giải cho bài toán đã được “âm thầm” kéo dài thêm 1 tháng,
tới thời điểm số Tạp chí này lên trang. Tòa soạn vẫn không nhận được một lời giải nào gửi đến
từ bạn đọc.

Ví dụ 1.12. Cho n (n ≥ 2) số thực bất kì có tổng bằng 0. Hỏi trong n số đó có ít nhất


bao nhiêu cặp có tổng không âm.

Ta gọi một cặp số thực có tổng không âm là một cặp "đẹp" và f (n) là số cặp đẹp ít nhất trong
n số thưc có tổng bằng 0. Ta có các nhận xét sau:
Nhận xét. Nếu có n số thực có tổng bằng k > 0 thì số cặp đẹp không nhỏ hơn f (n).
Nhận xét. Xét n số gồm n − 1 số −1 và số n − 1, khi đó ta thấy có n − 1 cặp đẹp, do đó
f (n) ≤ n − 1.

hu
Ta thấy f (2) = 1, f (3) = 1, f (4) = 3, f (5) = 3, f (6) = 5. Ta chứng minh f (n) = n − 1, ∀n ≥ 6
bằng cách chứng minh
f (n + 1) ≥ f (n) + 1 ∀n ≥ 6 .(1)
Thật vậy, ta xét n + 1 số thực có tổng bằng 0. T
t
• Nếu trong n + 1 số có một số bằng 0 thì ta loại số đó ra, khi đó n số còn lại có tổng bằng 0
Tấ

nên số cặp đẹp trong n số này là f (n). Vì trong n số này có ít nhất một số không a không
âm nên 0 và a tạo thêm một cặp đẹp, dẫn đến (1) đúng.
n

• Xét n + 1 số khác 0. Gọi A, B lần lượt là tập các số âm và tập các số dương trong n + 1
số đã cho. Ta có |A| + |B| = n + 1. Xét a = min A, a < 0, ta loại a thì n số còn lại có tổng
yễ

dương nên số cặp đẹp trong n số này không nhỏ hơn f (n).
gu

+) Nếu max B ≥ min A thì tồn tại b ∈ B để a + b ≥ 0, khi đó số cặp đẹp sẽ thêm 1, nên
(1) đúng.
2
N

+) Nếu max B < min A thì |A| < |B| và f (n + 1) = C|B| . Khi đó, nếu n + 1 = 2k + 1 thì
|B| ≥ k + 1 nên

2 k(k + 1)
f (n + 1) ≥ Ck+1 = ≥ 2k ≥ f (n) + 1 ∀k ≥ 3 (n ≥ 6).
2
Nếu n = 2k thì |B| ≥ k + 1 và chứng minh tương tự như trên.

Ví dụ 1.13. Cho một bảng ô vuông 2017 × 2019. Ta tô màu các ô vuông đơn vị của bảng
bởi màu đỏ. Hỏi ta có thể tô nhiều nhất bao nhiêu ô vuông sao cho mội hình vuông 2 × 2
cả bảng chứa nhiều nhất 2 ô được tô màu?

Ta gọi cách tô thỏa mãn yêu cầu bài toán là một cách tô đẹp. Gọi r là số ô nhiêu nhất được tô
màu của bảng (2m − 1) × (2n − 1) để được một cách tô đẹp (m,n ≥ 1). Giả sử m ≤ n, ta chứng
minh r = m(2n − 1).
Trước hết ta thấy cách tô các ô nằm ở hàng lẻ và không tô các ô ở các hàng chẵn là một cách tô
đẹp.
Bước tiếp theo ta chứng minh r ≤ m(2n − 1) bằng quy nạp theo m.
Với m = 1, ta có r = 2n − 1.

12
1. CỰC TRỊ TRONG TỔ HỢP

Với m = 2, ta xét bảng 3 × (2n − 1). Ta chia bảng này thành một bảng 2 × (2n − 1) và một
bảng 1 × (2n − 1). Khi đó, bảng 2 × (2n − 1) được chia thành n hình vuông 2 × 2, mà mỗi hình
vuông này có không quá 2 ô được tô, nên bảng 2 × (2n − 1) có không quá 2n ô được tô. Còn
bảng 1 × (2n − 1) có không quá 2n − 1 ô được tô. Từ đó, ta có

r ≤ 2n + 2n − 1 = 4n − 1. (1)

Dấu "=" ở (1) xảy ra khi bảng 3 × (2n − 1) có dạng như hình dưới

Ta thấy cách tô này không thỏa bài toán. Do đó r ≤ 4n − 2 = 2(2n − 1).


Giả sử bài toán đúng với m − 1, ta xét bảng (2m − 1) × (2n − 1). Ta chia bảng này thành hai

hu
bảng 2 × (2n − 1) (bảng A) và bảng (2m − 3) × (2n − 1) (bảng B). Khi đó

r = rA + rB ≤ 2n + (m − 1)(2n − 1) = 2mn − m − 1 = m(2n − 1) − 1. (2)

T
Giả sử đẳng thức ở (2) xảy ra, khi đó rA = 2n và rB = (m − 1)(2n − 1). Khi đó hàng đầu tiên của
bảng chứa n ô được tô màu. Ta chia bảng (2m − 2) × (2n − 1) thành m − 1 ô vuông 2 × (2n − 1),
t
mỗi bảng này có không qua 2n ô được tô, nên số ô được tô của bảng (2m − 2) × (2n − 1) không
Tấ

quá 2n(m − 1). Do đó

r ≤ 2n(m − 1) + n = 2mn − n ≤ 2mnm = m(2n − 1),


n

mẫu thuẫn với (2) xảy ra đẳng thức. Do đó đẳng thức ở (2) không xảy ra, hay r ≤ m(2n − 1).
yễ

Vậy bài toán được chứng minh.


gu

Ví dụ 1.14 (Chọn đội tuyển Nghệ an năm 2014-2015, ngày 2). Cho n ≥ 2 con
chim đậu trên đường tròn (O), mỗi điểm có nhiều nhất một con chim đậu. Hai con chim
N


đậu tại điểm Pi , Pj được gọi là trông thấy nhau nếu P i OPj ≤ 120 . Tìm theo n số nhỏ

nhất các cặp con chim trông thấy nhau.

Gọi f (n) là số nhỏ nhất"các cặp con


# chim trông thấy nhau.
2
(n − 1)
Ta chứng minh f (n) = .
4
Ta có f (2) = 0,"f (3) = 1#nên bài toán đúng với n = 2,n = 3.
(n − 1)2
 2
n
Giả sử f (n) = , n ≥ 2. Ta chứng minh f (n + 1) = .
4 4
Theo nguyên tắc cực hạn thì trong n + 1 con chim có một con chim trông thấy nhiều con chim
nhất, giả sử đó là con chim đậu ở điểm A (ta gọi tắt là A) và m là số con chim mà A nhìn thấy .
Trên đường tròn lấy hai điểm X,Y sao cho

AOX
÷ = XOY
÷ =Y÷OA = 120◦ .
_
Vì A trông thấy m con chim nên M con chim này nằm trên cung XAY tính cả hai điểm mút.
_
Do đó, trên cung nhỏ XY có n + 1 − (m + 1) = n − m con chim, và mỗi con chim này trông thấy

13
1. CỰC TRỊ TRONG TỔ HỢP

n − m − 1 con chim còn lại.


Suy ra số cặp chim trông thấy, trong đó có A là m và do tính chất của A nên ta có
n − 1 hni
m≥n−m−1⇒m≥ ≥ .
2 2
Số nhỏ nhất các cặp chim thấy nhau của n con chim còn lại (trừ A) là f (n) và rõ ràng các cặp
chim trông thấy nhau này khác với m cặp chim trông thấy nhau ở trên. Do đó, ta có
" #
(n − 1)2 hni
f (n + 1) = f (n) + m ≥ + .
4 2

Ta chứng minh " #


(n − 1)2 h n i  n2 
+ = . (1)
4 2 4
+) n = 2k thì
   2
1 2 n
V T (1) = k (k − 1) + + [k] = k (k − 1) + k = k = = V P (1).
4 4

hu
+) n = 2k + 1 thì    2
2 1 2 n
V T (1) = k + k + =k +k = = V P (1).
"
2
(n − 1)2
# T 4
t
Do đó, ta chứng minh được f (n) ≥ . Ta chỉ ra trường hợp có dấu “=”.
4
Tấ

_ hni

Trên đường tròn lấy hai điểm X,Y sao cho cung sđXY = 30 . Cho con chim đậu trên cung
  2
_ h n i n+1 _ _
nhỏ XY và n − = con chim còn lại đầu trên cung nhỏ X 0 Y 0 đối xứng với cung XY
n

2 2
_
yễ

_
qua tâm O. Khi đó, các con chim nằm trên hai cung XY và X 0 Y 0 trông thấy nhau , đồng thời
_ _
hai con chim bất kì nằm trên hai cung XY và X 0 Y 0 đều không trông thấy nhau. Do đó, số cặp
gu

chim trông thấy nhau là


    
h n i h n i n+1 n+1
N


−1 −1 "
2
#
2 2 2 2 (n − 1)
+ = .
2 2 4

Ví dụ 1.15 (Bài toán P80, Tạp chí Pi, tháng 10 năm 2017). Có 2017 người tham
dự một Festival quốc tế. Biết rằng, trong mỗi nhóm tùy ý gồm 7 người (trong 2017 người
vừa nêu) đều có tối đa 12 cặp quen nhau. Hỏi, trong Festical có thể có tối đa bao nhiêu
cặp quen nhau?

Dưới đây, khi nói Festival, ta hiểu là Festival mà ở, trong mỗi nhóm tùy ý 7 người (trong số
những người tham dự Festival) đều có không quá 12 cặp quen nhau.
• Trước hết, xét tính huống nhận được từ bài toán đã ra bằng cách thay 2017 bởi số nguyên
dương n ≥ 7 tùy ý; nghĩa là xét các Festival có n người tham dự (n ≥ 7).
Với Festival X có n người tham dự, kí hiệu fX (n) là số cặp quen nhau trong Festival đó.
Bằng quy nạp theo n ≥ 7, ta sẽ chứng minh
(
fX (n) ≤ k 2 nếu n = 2k, k ≥ 4,
(1)
fX (n) ≤ k(k + 1) nếu n = 2k + 1, k ≥ 3,

14
1. CỰC TRỊ TRONG TỔ HỢP

với Festival có n người tham dự.


Thật vậy:
+ Với n = 7 = 2 · 3 + 1, theo giả thiết của bài ra, ta có

fX (7) ≤ 12 = 3(3 + 1),

với mọi Festival có 7 người tham dự. Như vậy (1) đúng, với n = 7.
+ Giả sử (1) đã đúng với n = 2k − 1 = 2(k − 1) + 1, k ≥ 4; nghĩa là, ta có

fX (2k − 1) ≤ (k − 1)k, k ≥ 4,

với mọi Festival X có 2k − 1 người tham dự.


Giả sử ngược lại, tồn tại một Festival X có 2k người tham dự và trong Festival này:

fX (2k) ≥ k 2 + 1.

Khi đó, bằng cách cho một số thích hợp các cặp quen nhau trở thành không quen
nhau, ta sẽ có Festival Y có 2k người tham dự và

fY (2k) = k 2 + 1. (2)

hu
Dễ thấy, ở Festival Y , phải tồn tại một người quen với tối đa k người khác, vì nếu
ngược lại, một người quen với ít nhất k + 1 người khác thì

fY (2k) ≥
2k(k + 1
2
T
= k 2 + k ≥ k 2 + 4 (do k ≥ 4),
t
Tấ

mâu thuẫn với (2).


Tách người quen với tối đa k người khác ra khỏi Festival Y , ta sẽ có Festival Z có
2k − 1 người tham dự và
n

fZ (2k − 1) ≥ k 2 + 1 − k = (k − 1)k + 1,
yễ

trái với giả thiết quy nạp.


Vì vậy, ta có điều phải chứng minh.
gu

+ Giả sử (1) đúng với n = 2k, k ≥ 4; nghĩa là, ta có


N

fX (2k) ≤ k 2 , k ≥ 4,

với mọi Festival X có 2k người tham dự.


Xét n = 2k + 1. Ta cần chứng minh

fX (2k + 1) ≤ k(k + 1),

với mọi FestivalX có 2k + 1 người tham dự.


Giả sử ngược lại, tồn tại một Festival X có 2k + 1 người tham dự và

fX (2k + 1) ≥ k(k + 1) + 1 = k 2 + k + 1.

Khi đó bằng cách cho một số thích hợp các cặp quen nhau trở thành không quen
nhau, ta sẽ có Festival Y có 2k + 1 người tham dự và

fY (2k + 1) = k 2 + k + 1. (3)

Dễ thấy, ở Festival Y , phải tồn tại một người quen với tối đa k người khác, vì nếu
ngược lại, một người quen với ít nhất k + 1 người khác thì
(2k + 1)(k + 1)
fY (2k + 1) ≥ ≥= k 2 + k ≥ k 2 + 4
2
15
1. CỰC TRỊ TRONG TỔ HỢP

(do k ≥ 4 và fY (2k + 1) ∈ Z), mâu thuẫn với (3).


Tách người quan với tối đa k người khác ra khỏi Festival Y , ta sẽ có Festival Z có 2k
người tham dự và
fZ (2k) ≥ k 2 + k + 1 − k = k 2 + 1,
trái với giả thiết quy nạp.
Vì vậy, ta có điều cần chứng minh.
+ Vậy, tóm lại, theo nguyên lí quy nạp, (1) được chứng minh.
• Trở lại bài toán đã ra.
Ta có n = 2017 = 2 × 1008 + 1. Vì thế, theo kết quả đã chứng minh ở trên, ta có
fX (2017) ≤ 1008 × 1009 = 1017072,
với mọi Festival X có 2017 người tham dự.
Hơn nữa, xét một nhóm có 2017 người, mà 2017 người này có thể tách thành hai nhóm
con, một nhóm con gồm có 1008 người và một nhóm con gồm 1009 người, sao cho hai người
trong cùng một nhóm con không quen nhau và mỗi người ở nhóm con này đều quen với
tất cả mọi người ở nhóm con kia. Dễ thấy, nhóm người này cho ta một Festival X0 có 2017
người tham dự và có

hu
fX0 (2017) = 1008 × 1009 = 1017072.
Vậy số cặp quen tối đa có thể ở Festival nêu trong bài ra là 1017072.
Nhận xét.
T
1. Có thể thấy số 2017 không đóng vai trò gì trong quá trình giải bài toán. Vì thế, nghĩ tới viêc
t
thay số 2017 bởi số nguyên n ≥ 7 tùy ý, nhằm mở ra khả năng tiếp cận bài toán theo hướng
Tấ

quy nạp, là một điều khá tự nhiên.


Để giải bài toán khái quát (nhận được từ bài đã ra bằng cách thay 2017 bởi n ≥ 7), với fX (n)
là kí hiệu đã được định nghĩa trong lời giải nêu trên, ta cần tìm ra một biểu thức của n, gọi
n

là M (n), sao cho


yễ

fX (n) ≤ M (n),
với mọi Festival X thỏa mãn yêu cầu đề bài và với mọi số nguyên n ≥ 7.
gu

2. Giả thiết của bài toán cho ta biết giá trị của M (n) tại n = 7. Do đó, có thể nghĩ tới việc dự
đoán M (n) bằng suy luận quy nạp; nghĩa là, suy đoán M (n) dựa trên các giá trị của nó tại
N

một số giá trị ban đầu của n.


Đánh giá (1) trong lời giải là kết quả của việc suy đoán M (n) theo cách vừa nêu trên.

Ví dụ 1.16. Cho một bảng kích thước 2012 × 2012 được điền các số tự nhiên từ 1 đến
20122 theo quy tắc sau: Hàng thứ nhất ta điền các số từ 1 đến 2012 từ trái qua phải, ở
hàng thứ hai ta đánh các số từ 2013 đến 4024 tuwg phải qua trái, các hàng tiếp theo được
đánh theo kiểu zích zắc tương tự như trên. Hãy tìm các phủ kín bẳng trên bởi 1006 × 2012
quân cơ Domino sao cho tổng của tích các số trên mỗi quân cờ Domino lớn nhất.

Đặt A = 1,2, . . . ,20122 . Gọi ai , bi là hai số được ghi trên quân cờ Domino thứ i với ai , bi ∈
n
P
{1, 2, . . . , 1006 × 2012} ; i = 1, 1006 × 2012 và S = ai bi với n = 1006 × 2012. Ta cần tìm giá
i=1
trị nhỏ nhất của S. Vì
x2 + y 2 (x − y)2
xy = −
2 2
nên ta có: n n
1X 2 1X
S= (ai + b2i ) − (ai − bi )2 .
2 i=1 2 i=1

16
1. CỰC TRỊ TRONG TỔ HỢP

Mặt khác ai ,bi là các số tự nhiên khác nhau thuộc tập A nên
n
X 2n
X
(a2i + b2i ) = i2 và (ai − bi )2 ≥ 1.
i=1 i=1

Suy ra !
2n
1 X
S≤ i2 − n .
2 i=1

Đẳng thức xảy ra khi và chỉ khi ai , bi là hai số tự nhiên liên tiếp.
Vậy để S lớn nhất ta phủ các quân cơ Domino sao cho mỗi quân cờ chứa hai số tự nhiên liên
tiếp.

Ví dụ 1.17 (VMO 2012). Cho số nguyên dương n. Có n học sinh nam và n học sinh
nữ xếp thành một hàng ngang, theo thứ tự tùy ý. Mỗi học sinh (trong số 2n học sinh vừa
nêu) được cho một số kẹo bằng đúng số cách chọn ra hai học sinh khác giới với X và đứng
ở hai phía của X. Chứng minh rằng tổng số kẹo mà tất cả 2n học sinh nhận được không
1
vượt quá n(n2 − 1) .
3

hu
Gọi a1 , a2 , . . . , an và b1 , b2 , . . . , bn là vị trí của n nam và n nữ trên hàng.
Xét nam tại vị trí ai , ta thấy bên trái anh ta có ai − 1 vị trí, trong đó có i − 1 vị trí là nam, vậy
nên bên trái anh ta có ai − i nữ.
Tương tự, bên phải anh ta có n − (ai − i) nữ. T
t
Vậy nam tại ai được cho (ai − i)[n − (ai − i)] kẹo.
Tấ

Tương tự, nữ tại vị trí bi được cho (bi − i) [n − (bi − i)] kẹo.
Như vậy tổng số kẹo được cho bằng
n
n

X
S= {(ai − i)(n − (ai − i)) + (bi − i)(n − (bi − i))}
yễ

i=1
Xn
= {n(ai + bi ) − (a2i + b2i ) − 2ni − 2i2 + 2i(ai + bi ))}.
gu

i=1

Chú ý là
N

{a1 , . . . ,an ,b1 , . . . ,bn } = {1,2, . . . ,2n} ,


nên ta có
n 2n n 2n
X X 2n(2n + 1)(4n + 1) X X 2n(2n + 1)
(a2i + b2i ) = i2 = , (ai + bi ) = i= .
i=1 i=1
6 i=1 i=1
2

Ngoài ra
n n
X n(n + 1)(2n + 1) X n(n + 1)
i2 = , i= .
i=1
6 i=1
2

Thay vào biểu thức tính S, ta tìm được


n
X n(7n2 + 9n + 2)
S= 2i(ai + bi ) − .
i=1
3

Từ đó, ta đưa bài toán ban đầu về việc chứng minh bất đẳng thức:
n
X n(n + 1)(8n + 1)
T = i(ai + bi ) ≤ . (1)
i=1
6

17
1. CỰC TRỊ TRONG TỔ HỢP

Ta có:
an + bn ≤ 2n + 2n − 1 = 4n − 1
an + bn + an−1 + bn−1 ≤ 4n − 1 + 4n − 5
··················
an + bn + an−1 + bn−1 + · · · + a1 + b1 ≤ 4n − 1 + 4n − 5 + · · · + 3.

Áp dụng công thức khai triển tổng Abel, ta có


n
X
T = i(ai + bi ) = an + bn + (an + bn + an−1 + bn−1 ) + . . .
i=1
+ (an + bn + an−1 + bn−1 + . . . + a1 + b1 )
≤ 4n − 1 + (4n − 1 + 4n − 5) + · · · + (4n − 1 + 4n − 5 + · · · + 3)
n
X n(n + 1)(8n + 1)
= i(4i − 1) = .
i=1
6

Suy ra (1) đúng, hay bài toán được chứng minh.

hu
I. Bài tập
Bài 1.1 (Ninh Bình). Tìm số nguyên dương n nhỏ nhất sao cho nếu a1 , a2 , . . . , an là n số
T
phân biệt tùy ý được chọn từ tập X = {1; 2; . . . ; 17} ta luôn tìm được số nguyên dương k sao
cho phương trình ai − aj = k có ít nhất k nghiệm.
t
Bài 1.2. Cho tập hợp X = {2, 3, 4, . . . , 2014}. Tìm số nguyên dương nhỏ nhất n sao cho với
Tấ

mọi cách chọn n số nguyên tố cùng nhau từ tập X thì ta luôn chọn được ít nhất một số nguyên
tố.
n

Bài 1.3 (SP Hà Nội 2014-2015). Một bộ ba số nguyên dương (x; y; z) được gọi là một bộ
ba Pythagore nếu x2 + y 2 = z 2 . Tìm số k nhỏ nhất sao cho mỗi tập con gồm k phần tử của tập
yễ

S = {1,2,3, . . . ,25} luôn có ba phần tử tạo thành một bộ ba Pythagore.


Bài 1.4. Cho A là tập con của tập X = {1; 2; 3; . . . ; 2018} sao cho với hai số ai , aj ∈ A mà
gu

ai 6= aj thì tồn tại đúng một tam giác cân có độ dài các cạnh là ai , aj . Hỏi tập A có nhiều nhất
bao nhiêu phần tử ?
N

Bài 1.5. Cho A là tập con của tập các số nguyên dương thỏa mãn: với mọi x, y ∈ A mà x 6= y
xy
thì ta có |x − y| ≥ . Tìm số phần tử lớn nhất của tập A?
25
Bài 1.6 (Hà Tĩnh Ngày 1-2015). Cho tập S là tập hợp tất cả các bộ số, mỗi bộ gồm 2014 ×
2014 số thực bất kỳ thuộc đoạn [−1; 1] và có tổng bằng 0. Xét một bảng ô vuông kích thước
2014 × 2014. Tìm số dương k nhỏ nhất sao cho nếu điền bất kỳ một bộ số thuộc tập S vào bảng,
mỗi ô một số thì tồn tại ít nhất một hàng hoặc một cột có giá trị tuyệt đối của tổng các số trên
hàng đó hoặc trên cột đó không vượt quá k.
Bài 1.7. Giả sử S là tập con của tập A = {1; 2; 3; . . . ; 14; 15} thỏa mãn: tích 3 phần tử bất
kỳ thuộc S không là số chính phương. Tìm số phần tử lớn nhất của S.
Bài 1.8. Trong một giải đấu cờ vua, có n ≥ 5 đấu thủ tham gia (hai đấu thủ bất kỳ đấu  với
2
n
nhau tối đa một trận). Chứng minh rằng nếu số trận đấu đã diễn ra không nhỏ hơn +2
4
thì luôn tồn tại 5 đấu thủ A, B, C, D, E mà A đã đấu với cả 4 đấu thủ còn lại, ngoài ra B đã
đấu với C và D đã đấu với E. Khẳng định của bài toán còn đúng không nếu số trận đấu đã diễn
n2
ra nhỏ hơn + 2?
4

18
1. CỰC TRỊ TRONG TỔ HỢP

Bài 1.9. Cho T là một tập có n phần tử. Gọi X là tập các tập con của T gồm 3 phần tử mà
hai tập con bất kì có giao khác rỗng. Tìm số phần tử lớn nhất của tập X.
Bài 1.10. Cho n là số nguyên dương thỏa mãn các tính chất: nếu n quân domino được đặt trên
một bàn cờ 6 × 6 với mỗi domino chiếm 2 đơn vị diện tích vuông, thì luôn luôn có thể đặt thêm
một domino lên trên bàn mà không phải di chuyển bất kỳ domino nào khác. Xác định giá trị lớn
nhất của n.
Bài 1.11. Một tập A gồm các số nguyên dương được gọi là “đều”, nếu sau khi bỏ đi một phần
tử bất kì thì ta có thể chia các phần tử còn lại thành hai tập có tổng các phần tử bằng nhau.
Tìm số phần tử nhỏ nhất của một tập “đều”.
Bài 1.12. Cho X = {1; 2; . . . ; 2013} và A ⊂ X, |A| = n. Tìm n nhỏ nhất sao cho với mọi cách
a
chọn tập A ta luôn tìm được a, b ∈ A : = 3.
b
Bài 1.13 (China 2001). Tìm số nguyên dương m nhỏ nhất sao cho với mọi tập con A của tập
X = {1, 2, 3, . . . , 2001} đều có hai phần từ a, b (không nhất thiết phân biệt) thỏa a + b = 2n
với n là số nguyên dương nào đó.
Bài 1.14. Trong mặt phẳng cho 4 điểm phân biệt sao cho khoảng cách giữa hai điểm bất kì là
một số nguyên. Gọi S là tập hợp tất cả các khoảng cách nối hai điểm bất kì trong bốn điểm đã

hu
cho. Hỏi trong tập S có ít nhất bao nhiêu khoảng cách chia hết cho 3.
Bài 1.15 (THTT T10/447). Tìm số k nhỏ nhất để có thể tô màu các số 1, 2, · · · , 20 (mỗi số

T
tô một màu, hai số khác nhau có thể tô một màu) sao cho không có ba số nào tô cùng màu lập
thành một cấp số cộng.
t
Bài 1.16 (P33). Hỏi trên mặt phẳng, có thể vẽ được nhiều nhất bao nhiêu tam giác mà hai
Tấ

tam giác bất kì (trong số đó) đều có đúng một đỉnh chung và không có điểm nào là đỉnh chung
của tất cả các tam giác.
Bài 1.17. Cho tập X = {1, 2, · · · , 2018}. Xét tập A là tập con của X có tính chất: Với ba số
n

bất kì thuộc A (có thể trùng nhau) thì ba số đó luôn là độ dài ba cạnh của một tam giác không
yễ

tù. Tìm số phần tử lớn nhất có thể có của tập A.


Bài 1.18. Mỗi điểm trong số 2009 điểm cho trước trên mặt phẳng được tô bởi một trong hai
gu

màu: xanh hoặc đỏ. Biết rằng mỗi đường tròn đơn vị có tâm màu xanh đều đi qua đúng hai điểm
đỏ. Hỏi có nhiều nhất bao nhiêu điểm xanh?
N

Bài 1.19. Một CLB du khảo có n người trong năm vừa qua, nhóm tổ chức được 6 chuyến du
khảo, mỗi một chuyến có 5 người tham gia. Một thành viên CLB nhận xét là: Hai chuyến du
khảo bất kì có không quá 2 thành viên chung. Hỏi CLB có ít nhất bao nhiêu người.
Bài 1.20. Trong một cuộc thi có 11 thí sinh tham gia giải 9 bài toán. Hai thí sinh bất kì giải
chung với nhau không quá 1 bài. Tìm k lớn nhất để mọi bài toán có ít nhất k thí sinh giải
được.
Bài 1.21 (Lào Cai 2017-2018). Cho đa giác lồi có 17 đỉnh A1 A2 A3 ...A17 và với hai đỉnh Ai ,Aj
−−−→
bất kỳ trong số các đỉnh của đa giác, ta định hướng cho đoạn thẳng nối chúng để có vectơ: Ai Aj
−−−→
hoặc Aj Ai . Sau khi thực hiện với mọi cặp đỉnh, gọi S là số tam giác có tổng các vectơ đặt trên


3 cạnh là 0 .

a) Tìm giá trị nhỏ nhất của S.

b) Tìm giá trị lớn nhất của S.

Bài 1.22 (PTNK 2017-2018). Gọi S là tập con của tập X = {1, 2, · · · , 2017} sao cho S
không chứa hai phần tử mà phần tử này chia hết cho phần tử kia và cũng không chứa hai phần
tử nguyên tố cùng nhau. Tìm số phần tử lớn nhất của S.

19
1. CỰC TRỊ TRONG TỔ HỢP

Bài 1.23 (Thanh Hóa 2017 – 2018). Cho số nguyên tố p. Trên bàn cờ hình vuông có cạnh
p2 + p + 1, tìm số lớn nhất các ô có thể tô màu sao cho không tồn tại hình chữ nhật có 4 đỉnh
cùng màu và các cạnh song song với các cạnh của hình chữ nhật.
Bài 1.24. Cho n ≥ 2 là một số nguyên. Tập S gồm các đường chéo của 4n − 1 giác lồi được
phân hoạch thành k tập S1 ,S2 , . . . ,Sk sao cho với hai tập Si , Sj bất kì thì luôn có một đường
chéo trong Si cắt một đường chéo trong Sj . Tìm giá trị lớn nhất của k.
Bài 1.25. Một lớp học có n học sinh. Vào dịp Tết, một số bạn học sinh của lớp có gọi điện chúc
mừng nhau. Biết rằng mỗi học sinh gọi điện chúc mừng cho không quá ba bạn, với hai bạn bất
kỳ thì có không quá một người gọi điện chúc mừng cho người còn lại và với ba bạn bất kỳ thì
luôn có hai người mà trong đó có một người gọi điện chúc mừng cho người còn lại. Tìm giá trị
lớn nhất có thể của n.
Bài 1.26. Cho tập S = {1; 2; . . . ; 2017} và A1 ,A2 , . . . ,Ak là các tập con của S sao cho với mọi
1 ≤ i < j ≤ k có đúng một trong các tập Ai ∩ Aj , A0i ∩ Aj ,Ai ∩ A0j ,A0j ∩ A0j là tập rỗng. Tìm giá
trị lớn nhất có thể có của k. (Với A ⊂ S,A0 kí hiệu là phần bù của tập A trong S).
Bài 1.27. Cho một bảng vuông cỡ 2n × 2n (với n là số nguyên dương). Ta gọi một đường đi
chéo là một ô vuông hoặc một tập hợp các ô vuông phân biệt C1 , C2 , . . . , Ck (với k là số nguyên
dương) sao cho hai ô Ci , Ci+1 có đúng một đỉnh chung (i = 1; k − 1). Hai đường đi chéo được gọi

hu
là rời nhau nếu chúng không có ô vuông chung. Hỏi bảng vuông đã cho có thể phân hoạch thành
ít nhất bao nhiêu đường đi chéo rời nhau?

T
Bài 1.28. Có 2000 học sinh tham gia một cuộc thi trắc nghiệm gồm 5 câu hỏi; mỗi câu có 4
phương án trả lời, mỗi học sinh chỉ được phép chọn 1 trong 4 phương án tương ứng để trả lời
cho câu hỏi. Tìm số tự nhiên n bé nhất sao cho các học sinh có thể làm bài thi theo cách nào đó
t
Tấ

mà cứ n học sinh thì tìm được bốn học sinh (trong số n học sinh này) để hai hai học sinh nào
trong số bốn học sinh đó cũng có bài làm khác nhau ở ít nhất là hai câu hỏi.
Bài 1.29. Có n ≥ 2 đội bóng tham gia một giải đấu bóng đá. Giải đấu được tổ chức theo hình
n

thức thi đấu vòng tròn: 2 đội bóng bất kì thi đấu với nhau đúng 1 trận. Cách tính điểm như
sau: sau mỗi trận, đội thắng được 3 điểm, đội thua được 0 điểm, còn nếu hòa thì cả 2 đội được
yễ

1 điểm. Sau khi giải đấu kết thúc, người ta thấy rằng không có 2 đội nào có cùng số điểm. Hỏi
khoảng cách tối thiểu đội đứng đầu và đội đứng cuối là bao nhiêu?
gu

Bài 1.30. Cho một bảng ô vuông kích thước 2017 × 2017, người ta điền vào một ô vuông của
bảng ô vuông đó các số 1,2,...,20172 sao cho mỗi số được điền vào mỗi ô vuông của bảng đúng
N

một lần.
a) Chứng minh rằng tồn tại hai số được điền vào hai ô vuông lân cận có hiệu số không lớn
hơn 2017 (hai ô vuông lân cận là hai ô vuông có cạnh chung).

b) Tìm số nguyên dương k bé nhất sao cho tồn tại một cách điền các số vào các ô vuông của
bảng mà hiệu hai số ở hai ô vuông lân cận bất kì không lớn hơn k.
Bài 1.31. Với hai số có 5 chữ số a = a1 a2 a3 a4 a5 ; b = b1 b2 b3 b4 b5 , ta gọi số i lớn nhất mà ai 6= bi
P cách giữa a và b, kí hiệu là d(a, b). Xét hoán vị H của tất cả các số có 5 chữ số, đặt
là khoảng
S = d(a, b). Trong đó, lấy tổng theo tất cả các cặp (a, b) mà a, b nằm cạnh nhau trong H.
Tìm giá trị nhỏ nhất của S.
Bài 1.32. Các số nguyên dương từ 1 đến 9 được viết trong mỗi hình vuông của bảng vuông
3 × 3, mỗi ô chứa đúng một số và các ô khác nhau chứa các số khác nhau. Ta thực hiện thao tác
như sau: Lấy 1 hàng hoặc 1 cột và thay các số a, b, c theo thứ tự trong hàng hoặc cột đó bởi các
số không âm a − x,b − x,c + x hoặc a + x, b − x, c − x, với x là số dương và có thể thay đổi theo
từng thao tác.
a) Tồn tại hay không một chuỗi các thao tác sao cho sau chuỗi thao tác đó, tất cả các số trong
bảng vuông đều bằng nhau nếu bảng ban đầu được cho như sau:

20
1. CỰC TRỊ TRONG TỔ HỢP

a) b)
1 2 3 2 8 5
4 5 6 9 3 4
7 8 9 6 7 1

b) Xác định số M lớn nhất sao cho sau 1 số bước, tất cả các ô trong bảng đều chứa M .

Bài 1.33. Xét các cách viết các số 12 , 22 , . . . , 82 trên 8 đỉnh của một hình lập phương, mỗi đỉnh
viết một số và hai đỉnh khác nhau viết hai số khác nhau. Trong một cách viết, với mỗi cạnh của
hình lập phương ta lập một số là tích hai số ở hai đầu mút của cạnh đó, gọi S là tổng của 12 số
như vậy. Tìm giá trị lớn nhất của S.
Bài 1.34. Thầy giáo ghi 2017 số thực khác nhau vào 2017 thẻ, mỗi thẻ một số, sau đó đặt xấp
các thẻ này lên bàn liên tiếp nhau. Một học sinh A, mỗi lần được phép hỏi Thầy giáo tập hợp
gồm ba số được ghi trên ba thẻ bất kì. Thầy giáo sẽ thông báo tập hợp gồm ba số ấy cho bạn
A (nhưng Thầy giáo không nói rõ số được ghi trên từng thẻ trong ba thẻ này). Học sinh A cần
phải hỏi Thầy giáo ít nhất bao nhiêu lần để chắc chắn biết hết các số được ghi trên từng thẻ
trong 2017 thẻ đã cho.
Bài 1.35. Cho n ≥ 3 là số nguyên dương. Các điểm A1 , A2 , · · · , An cùng thuộc một đường

hu
tròn. Có tối đa bao nhiêu tam giác nhọn có đỉnh là 3 trong số các đỉnh trên nếu:

a) n = 4

b) n = 2017. T
t
Bài 1.36. Cho một bảng ô vuông kích thước 2017 × 2017, người ta điền vào một ô vuông của
Tấ

bảng ô vuông đó các số 1,2,...,20172 sao cho mỗi số được điền vào mỗi ô vuông của bảng đúng
một lần.
n

a) Chứng minh rằng tồn tại hai số được điền vào hai ô vuông lân cận có hiệu số không lớn
hơn 2017 (hai ô vuông lân cận là hai ô vuông có cạnh chung).
yễ

b) Tìm số nguyên dương k bé nhất sao cho tồn tại một cách điền các số vào các ô vuông của
gu

bảng mà hiệu hai số ở hai ô vuông lân cận bất kì không lớn hơn k.

Bài 1.37. Xét các cách viết các số 12 , 22 , . . . , 82 trên 8 đỉnh của một hình lập phương, mỗi đỉnh
N

viết một số và hai đỉnh khác nhau viết hai số khác nhau. Trong một cách viết, với mỗi cạnh của
hình lập phương ta lập một số là tích hai số ở hai đầu mút của cạnh đó, gọi S là tổng của 12 số
như vậy. Tìm giá trị lớn nhất của S.
Bài 1.38 (KHTN -2015). Cho bảng ô vuông n × n với n ∈ N∗ và số nguyên k ≤ n. Điền vào
các ô trong bảng n × n các số thực thuộc đoạn [−1; 1] sao cho tổng các số trên mỗi bảng con
k × k đều bằng 0. Tìm giá trị lớn nhất của tổng tất cả các số trên bảng.
Bài 1.39. Cô giáo có tất cả 2020 viên kẹo gồm 20 loại kẹo khác nhau, mỗi loại ít nhất có 2 viên
kẹo. Cô chia hết kẹo cho các học sinh của mình, mỗi người một số viên kẹo và không có học sinh
nào nhận được nhiều hơn một viên kẹo ở một loại kẹo. Cô yêu cầu hai học sinh khác nhau bất kì
so sánh các viên kẹo mình nhận được và viết số loại kẹo mà cả hai cùng có lên bảng. Biết rằng
mỗi cặp học sinh bất kì đều được lên bảng đúng một lần. Gọi tổng các số được viết lên bảng là
M.

a) Xác định giá trị nhỏ nhất của M .

b) Với giả thiết tương tự nhưng thay 20 loại kẹo khác nhau bởi 19 loại kẹo khác nhau, hãy
tìm giá trị nhỏ nhất của M trong trường hợp tương ứng này.

21
1. CỰC TRỊ TRONG TỔ HỢP

Bài 1.40. Tại một hội nghị khoa học có 100 đại biểu tham dự. Người ta nhận thấy rằng không
có 3 đại biểu nào đôi một quen nhau. Biết rằng tồn tại số nguyên dương n sao cho không có đại
biểu nào quen quá n đại biểu khác và với mọi k,1 ≤ k ≤ n có ít nhất một đại biểu quen đúng k
đại biểu khác. Hãy tìm giá trị lớn nhất của n.
Bài 1.41. Trong mặt phẳng cho tập hợp A gồm 20182 điểm phân biệt được đánh số từ 1 đến
20182 sao cho ba điểm bất kỳ nào đó trong chúng không thẳng hàng. Một tứ giác được gọi là
"đẹp" nếu các đỉnh của nó thuộc A và được đánh số bằng 4 số thỏa mãn một trong hai điều kiện
sau:
a) Là các số tự nhiên cách nhau 2018 đơn vị, tức là 4 số lập thành cấp số cộng có công sai
2018.
b) Là 4 số tự nhiên liên tiếp và nếu trong đó có chứa số chia hết cho 2018 thì số đó phải là số
lớn nhất.
Nối tất cả các điểm thuộc tập hợp A lại với nhau sao cho điểm nào thuộc A cũng thuộc đúng
một tứ giác. Tìm số lớn nhất tứ giác "đẹp" được tạo thành.
Bài 1.42. Tại một Hội khỏe Phù Đổng gồm có n vận động viên tham gia vào 12 môn thi đấu,
mỗi môn thi đấu có 24 vận động viên. Biết rằng hai thành viên bất kì tham gia chung không

hu
quá một môn thi. Tìm giá trị nhỏ nhất của n.
Bài 1.43 (P4, Tạp chí Pi tháng 01 năm 2017). Có 100 chú gấu vào rừng hái quả. Chú gấu

T
nhỏ nhất hái được 1 quả, chú gấu tiếp theo hái được 2 quả, chú gấu thứ ba hái được 3 quả, cứ
thế cho đến chú gấu lớn nhất hái được 100 quả. Các chú gấu gặp con sói già. Sói đề xuất sẽ giúp
chúng chia quả hái được một cách “công bằng”. Sói đưa ra quy trình như sau: Sói sẽ chọn hai
t
chú gấu bất kì, gộp số quả hái được lại và chia đều, nhưng nếu có 1 quả lẻ thì sói sẽ ăn quả đó.
Tấ

Sói sẽ làm như vậy cho đến khi tất cả các chú gấu có số quả bằng nhau. Hỏi sói có thể ăn nhiều
nhất bao nhiêu quả?
Bài 1.44 (P39, Tạp chí Pi, số tháng 3, năm 2018). Kí hiệu S là tập hợp gồm 2017 số
n

nguyên dương đầu tiên. Hỏi có nhiều nhất bao nhiêu phần tử của S là những số hạng của
yễ

cùng một cấp số nhân tăng, có các số hạng là số thực và công bội lớn hơn 1?
Bài 1.45 (P42, Tạp chí Pi, tháng 4 năm 2017). Hỏi có thể chọn ra tối đa bao nhiêu số từ
gu

2017 số nguyên dương đầu tiên, sao cho tổng lập phương của hai số bất kì trong các số được
chọn ra là một số không chia hết cho 4?
N

Bài 1.46 (P104, Tạp chí Pi, tháng 10 năm 2017). Tìm số nguyên n ≥ 3 nhỏ nhất có tính
chất: Với mọi cách tô n điểm A1 , A2 , . . . ,An , đôi một phân biệt và theo thứ tự đó nằm cách đều
nhau trên một đường thẳng, bởi hai màu, luôn tồn tại ba điểm Ai ,Aj ,A2j−i (1 ≤ i < 2j − i ≤ n)
được tô cùng màu.
Bài 1.47 (TST VN 2007, Pro5). Cho A là tập con gồm 2007 phần tử của tập {1, 2, . . . , 4014}
thỏa mãn với mọi a, b ∈ A thì a không chia hết cho b. Kí hiệu mA là phầ tử nhỏ nhất của tập
A. Tìm giá trị nhỏ nhất của mA với tập A thỏa mãn các điều kiện nêu trên.
Bài 1.48 (ST V N 2008 Pro 3). Cho số nguyên n > 3. Kí hiệu T là tập hợp gồm n số
nguyên dương đầu tiên. Một tập con S của T được gọi là tập khuyết trong T nếu S có tính chất:
n
Tồn tại số nguyên dương c không vượt quá sao cho với s1 ,s2 là hai số bất kì thuộc S ta luôn
2
có |s1 − s2 | =
6 c. Hỏi tập khuyết trong T có thể có tối đa bao nhiêu phần tử ?
Bài 1.49 (VN TST 2012). Trên một cánh đồng hình chữ nhật kích thước m × n ô vuông gồm
m hàng và n cột, người ta đặt một số máy bơm nước vào các ô vuông. Biết rằng mỗi máy bơm
nước có thể tưới nước không những cho ô vuông chứa nó và các ô vuông có chung cạnh với ô
đó mà còn có thể tưới cho các ô vuông cùng cột với nó và cách nó đúng một ô vuông. Tìm số
nhỏ nhất các máy bơm nước cần đặt để các máy bơm đó có thể tưới hết cả cánh đồng trong hai
trường hợp:

22
1. CỰC TRỊ TRONG TỔ HỢP

a) m = 4. b) m = 3.

Bài 1.50 (VN TST 2013). Cho một khối lập phương 2n × 2n × 2n gồm 8n3 ô vuông đơn vị
màu trắng. An và Bình chơi một trò chơi. An chọn một số dải 1 × 1 × n sao cho hai dải bất kì
không có chung đỉnh hoặc cạnh rồi đổi tất cả các ô sang màu đen. Bình thì được chọn một số ô
bất kì của hình lập phương rồi hỏi An các ô này màu gì. Hỏi Bình phải chọn ít nhất bao nhiêu
ô để với mọi câu trả lời của An thì Bình luôn xác định được những ô nào là màu đen?
Thật vậy, gọi Sn là tập hợp các khối mà Bình cần phải chọn để hỏi An và với mỗi ô u được
Bình chọn, đặt Ru là hợp của các ô thuộc ba dải ngang, dọc, chéo đi qua u. Ta thấy rằng:

hu
T
t
Tấ
n
yễ

Do điều kiện hai dãy được chọn bất kì không chung cạnh và đỉnh nên với mỗi câu trả lời về màu
gu

cho mỗi ô u mà Bình chọn. Ta thấy rằng nếu gọi Ru là hợp của các ô được chọn thì khi ô đó màu
đen, sẽ có đúng một trong ba dải ngang, dọc, chéo đi qua ô đó được tô màu đen. Trong trường
N

hợp này, ta cần chọn thêm một số ô thuộc Ru để biết chính xác dải đó. Nếu chỉ chọn thêm một
ô thôi thì khi An trả lời ô đó màu trắng, Bình sẽ không xác định được dải nào trong hai dải còn
lại được tô đen. Do đó, Bình phải chọn thêm ít nhất hai ô nữa trong Ru mới có khả năng trả lời
được. Thêm vào đó, hai ô đó phải thuộc hai dải khác nhau vì nếu chúng cùng thuộc một dải thì
cũng tương tự như trường hợp chọn một ô nêu trên.
Với nhận xét này, ta gán cho mỗi ô u của hình lập phương một bộ số (a; b; c) với định nghĩa
như sau:

• a = 2 nếu dải hình hộp theo chiều ngang đi qua u không có thêm điểm nào thuộc Sn và
a = 1 nếu ngược lại.

• b = 2 nếu dải hình hộp theo chiều dọc đi qua u không có thêm điểm nào thuộc Sn và b = 1
nếu ngược lại.

• c = 2 nếu dải hình hộp theo chiều chéo đi qua u không có thêm điểm nào thuộc Sn và c = 1
nếu ngược lại.

Khi đó, có hai trong ba số a, b, c có giá trị bằng 1 và số còn lại không vượt quá 2 nên

a + b + c ≤ 4.

23
1. CỰC TRỊ TRONG TỔ HỢP

Đặt T là tổng các số dùng để gán cho các ô của hình lập phương. Khi đó, ta có
X
T = (a + b + c) ≤ 4|Sn |.
u∈Sn

Mặt khác, với mỗi dãy 1 × 1 × n của khối lập phương (theo cả ba chiều) đều có ít nhất một khối
thuộc tập hợp Sn vì nếu không, Bình sẽ không có thông tin gì về dải đó và trong trường hợp An
trả lời rằng tất cả các ô được Bình chọn đều được tô màu trắng thì Bình sẽ không biết được dải
còn lại đó có được tô màu đen hay không.
Dễ thấy rằng có tất cả (2n)2 dải 1 × 1 × 2n nằm ngang và tất cả các dải này sẽ đóng góp ít nhất
2(2n)2 đơn vị vào T (đóng góp vào các số a theo định nghĩa như trên).
Tương tự với (2n)2 dải 1 × 1 × 2n dọc và (2n)2 dải 1 × 1 × 2n chéo nên ta suy ra

T ≥ 3 · 2(2n)2 = 24n2 ⇒ 4|Sn | ≥ 24n2 ⇒ |Sn | ≥ 6n2 .

Đến đây, ta suy ra hai điều sau:


- Trong hình lập phương 2 × 2 × 2, Bình cần chọn ít nhất 6 ô.
- Trong hình lập phương 10 × 10 × 10, Bình cần chọn ít nhất 150 ô.
Ta sẽ chỉ ra cách tô màu 6 ô thỏa mãn đề bài và từ đó chỉ ra cách xây dựng cho hình lập phương

hu
10 × 10 × 10 đã cho. (Trên thực tế, ta hoàn toàn có thể xây dựng cho trường hợp tổng quát cho
hình lập phương 2n × 2n × 2n).
Thật vậy, trong hình lập phương 2 × 2 × 2, trừ hai ô nào đó đối xứng nhau qua tâm, Bình chọn
6 ô còn lại như hình bên dưới.
Ta sẽ chứng minh cách chọn này thỏa mãn yêu cầu. T
t
Tấ
n
yễ
gu
N

Dễ thấy rằng trong hình lập phương 2 × 2 × 2, chỉ có không quá 1 dải được tô màu nên hoặc
không có ô nào hoặc có 2 ô của hình lập phương được tô màu và 2 ô đó phải thuộc cùng 1 dải.
Do đó, trong 6 ô được chọn, ta có 3 trường hợp:
- Nếu không có ô nào được tô đen thì cả hình lập phương không có.
- Nếu có đúng 1 ô được tô đen thì ô đen còn lại sẽ thuộc trong 2 ô không được chọn có cùng 1
dải với ô đen đã biết.
- Nếu có 2 ô được tô đen thì đó chính là tất cả các ô đen của hình lập phương.
Do đó, cách chọn này với hình lập phương 2 × 2 × 2 thỏa mãn điều kiện đề bài.
Tiếp theo, ta xây dựng cho hình lập phương 10 × 10 × 10 như sau:
Ta chia hình lập phương thành 5 lớp 10 × 10 × 2 và ta chia nó thành 25 phần, mỗi phần là một
hình lập phương 2 × 2 × 2 rồi đánh số như hình bên dưới:

24
1. CỰC TRỊ TRONG TỔ HỢP

Ở lớp thứ i với i = 1, 2, 3, 4, 5, ta chọn các khối được đánh số i và với mỗi khối đó, ta bỏ đi 2 ô
bất kì đối xứng nhau qua tâm như cách đã nêu ở trên rồi chọn 6 còn lại.
Dễ thấy rằng với cách tô như vậy, ta chiếu các hình lập phương đã được chọn xuống một mặt
nào đó thì các hình chiếu sẽ phủ kín mặt đó. Điều này có nghĩa là với một dải bất kì mà An đã

hu
chọn thì nó đều đi qua một trong các hình lập phương 2 × 2 × 2 mà Bình chọn như trên. Khi
đó, như đã chứng minh ở trên, ta sẽ xác định được rằng dải đó có được tô màu hay không, tức
là xác định được màu của tất cả các ô được tô màu đen của hình lập phương ban đầu. Do đó,
cách chọn các ô này thỏa mãn đề bài.
Vậy số ô ít nhất mà Bình cần chọn là 150. T
t
Tấ
n
yễ
gu
N

25
1. CỰC TRỊ TRONG TỔ HỢP

ĐÁP SỐ VÀ HƯỚNG DẪN GIẢI


Câu 1.1. Xét tập A = {1; 3; 6; 9; 13; 17} ⊂ X.
Ta thấy với mọi ai , aj ∈ A thì phương trình ai − aj = k có ít hơn k nghiệm. Suy ra n ≥ 7.
Ta chứng minh n = 7 thỏa mãn.
Xét A = {1 ≤ a1 < a2 < · · · < a7 ≤ 17} là tập con bất kì của tập X. Giả sử phương trình
ai − aj = k có ít hơn k nghiệm. Suy ra không có cặp (ai , aj ) để ai − aj = 1, có nhiều nhất một
cặp (ai ; aj ) : ai − aj = 2, có nhiều nhất hai cặp (ai ; aj ) : ai − aj = 3 và có nhiều nhất 3 cặp
(ai ; aj ) : ai − aj = 4.
Khi đó
X6
a7 − a1 = (ai+1 − ai ) ≥ 2 + 2 · 3 + 3 · 4 = 20.
i=1

Điều này vô lí, do a7 ≤ 17.


Vậy nmin = 7. 

Câu 1.2. Xét tập

A = 22 , 32 , 52 , 72 , 112 , 132 , 172 , 192 , 232 , 292 , 312 , 372 , 412 , 432


hu
ta có A ⊂ X, |A| = 14, các phần tử trong A đôi một nguyên tố cũng nhau và trong tập A không
chứa số nguyên tố nào.
Do đó n ≥ 15. T
Bây giờ ta lấy 15 phần tử bất kì từ tập X mà hai phần tử đôi một nguyên tố cùng nhau. Giả sử các
t
số lấy ra là a1 < a2 < · · · < a15 . Giả sử ai là hợp số với mọi i = 1, 15. Gọi pi là ước nguyên tố nhỏ
Tấ

nhất của ai (i = 1, 15). Vì các số a1 , a2 , . . . , a15 đôi một nguyên tố cùng nhau nên p1 , p2 , . . . , p15
là các số nguyên tố phân biệt. Suy ra p15 ≥ 47, do đó a15 ≥ p215 ≥ 472 = 2209 > 2014 (vô lí).
Do đó trong các số a1 , a2 , . . . , a15 có ít nhất một số nguyên tố.
n

Vậy nmin = 15. 


yễ

Câu 1.3. Xét các bộ (x; y; z) ∈ S 3 là bộ Pythagore.


Khi đó (x; y; z) là một trong các bộ sau
gu

(3; 4; 5), (6; 8; 10), (9; 12; 15), (12; 16; 20), (15; 20; 25), (5; 12; 13), (8; 15; 17),
(7; 24; 25).
N

Các bộ trên được lập thành từ các số

{3, 4, 5, 6, 7, 8, 9, 10, 12, 13, 15, 16, 20, 24, 25} .

Xét tập

A = {3; 4; 6; 10; 9; 20; 16; 13; 17; 7; 24; 1; 2; 11; 14; 17; 18; 19; 21; 22; 23} .

Ta có |A| = 21 và rõ ràng trong tập A không có bộ ba số Pythagore.


Suy ra kmin ≥ 22. 

Câu 1.4. Xét hai số nguyên dương a, b ∈ A với a < b. Do có duy nhất một tam giác cân có độ
dài các cạnh là a, b nên a, b, b là độ dài ba cạnh đó, còn a, a, b không là độ dài ba cạnh của tam
giác, hay 2a ≤ b. Giả sử A = {a1 , a2 , . . . , an } với a1 < a2 < · · · < an . Khi đó ta có

2018 ≥ an ≥ 2an−1 ≥ · · · ≥ 2n−1 a1 ≥ 2n−1 .

Suy ra n − 1 ≤ 10 hay n ≤ 11.


Xét tập A = {1; 2; 4; . . . ; 1024}, ta có |A| = 11 và tập A thỏa yêu cầu bài toán.
Vậy max |A| = 11. 

26
1. CỰC TRỊ TRONG TỔ HỢP

Câu 1.5. Giả sử A = {a1 < a2 < · · · < an }. Khi đó ta có


ai ai+1 1 1 1
ai+1 − ai ≥ ⇒ − ≥ .
25 ai ai+1 25
Suy ra
1 1 n−1 25
− ≥ ⇒n−1< ≤ 25 ⇒ n ≤ 25.
a1 an 25 a1
25 25 25 25
Tương tự n − 2 < ≤ ⇒ n ≤ 14, . . ., n − 5 < ⇒ n ≤ 9, n − 6 < ⇒ n ≤ 10. Ta chứng
a2 2 a5 a6
minh n = 9 thỏa mãn.
xy
Trước hết ta thấy nếu xy ≤ 25 thì |x − y| ≥ , do đó 1, 2, 3, 4, 5 ∈ A.
25
6·5
Do |6−5| < nên 6 6∈ A. Tiếp tục ta có 7, 10, 17, 54 ∈ A. Do đó A = {1; 2; 3; 4; 5; 7; 10; 17; 54}
25
thỏa yêu cầu bài toán.
Vậy max |A| = 9. 
Câu 1.6. Chia bảng ô vuông thành 4 phần bằng nhau như hình vẽ. Chọn một bộ số trong S
điền vào các ô vuông sao cho trong phần 1 ta điền số 1; trong phần 3 ta điền số −1; các ô vuông
trong phần 2 và 4 ta điền số 0.

hu
Với cách điền số này giá trị tuyệt đối của tổng các số trên mỗi hàng hoặc trên mỗi cột đều bằng
1007.
Ta sẽ chứng minh k = 1007 là số nhỏ nhất thỏa mãn yêu cầu đề bài.

T
t
1 2
Tấ
n

3 4
yễ
gu

Giả sử tồn tại cách điền số vào bảng ô vuông sao cho các hàng và các cột của bảng đều có giá
trị tuyệt đối của tổng các số trên đó lớn hơn 1007. (1)
Nhận xét.
N

+ Nếu có bảng ô vuông thỏa mãn (1) thì đổi dấu tất cả các số của bảng ta được bảng mới cũng
thỏa mãn.
+ Nếu hoán vị các hàng thì tổng các số trên mỗi cột không thay đổi, nếu hoán vị các cột thì
tổng các số trên mỗi hàng không thay đổi.
Xét một bảng thỏa mãn (1)
Vì tổng các số trên bảng bằng 0 nên tồn tại ít nhất 1007 hàng có tổng các số trên mỗi hàng đều
không âm hoặc đều không dương. Không mất tính tổng quát giả sử có ít nhất 1007 hàng có tổng
các số trên mỗi hàng không âm (nếu ngược lại ta đổi dấu tất cả các số hạng). Hoán vị các hàng
sao cho các hàng đó đều thuộc phần 1, phần 2.
Tương tự tồn tại ít nhất 1007 cột có tổng các số trên mỗi cột đều không âm hoặc không dương.
Hoán vị các cột sao cho các các cột thuộc phần 2 và 3 có tổng các số trên mỗi cột đều không âm
hoặc đều không dương.
Gọi Si là tổng các số thuộc phần i, ta có :
S1 + S2 > 1007⇒ S3 + S4 < −10072 . (3)

S2 + S3 > 10072 hoặc S2 + S3 < −10072 .

27
1. CỰC TRỊ TRONG TỔ HỢP

• S2 + S3 > 10072 , kết hợp với (3) ta có S2 − S4 > 2.10072 . (4)


Mặt khác vì các số thuộc [−1; 1] nên

−10072 ≤ S2 ,S4 ≤ 10072 ⇒ S2 − S4 ≤ 2.10072

Điều này mâu thuẫn với (4).

• S2 + S3 < −10072 , kết hợp với (2) ta có S1 − S3 > 2.10072 . (5)


Mặt khác vì các số thuộc [−1; 1] nên

−10072 ≤ S1 ,S3 ≤ 10072 ⇒ S1 − S3 ≤ 2.10072

Điều này mâu thuẫn với (5).

Bài toán được chứng minh. 

Câu 1.7. Xét tập S0 = {3; 4; 5; 6; 7; 9; 10; 11; 13; 14}, ta có |S0 | = 10 và trong S0 không
có tích ba số là một số chính phương. Do đó |S| ≥ 11.
Xét tập S ⊂ A và |S| = 11. Ta xét các trường hợp sau

hu
• 2 ∈ S, khi đó nếu S một cặp trong các cặp sau

{5; 10}, {7; 14}, {3; 6}, {4; 8}

T
thì cặp đó với số 2 tạo thành ba số có tích là số chính phương. Nếu trong các cặp trên mỗi
cặp chỉ có nhiều nhất một số, do |S| = 11 nên mỗi cặp trên có đúng một số thuộc S và các
t
số 1, 2, 9, 11, 12, 13, 15 ∈ S. Do đó 1, 4, 9 ∈ S hoặc 2, 8, 1 ∈ S. Khi đó tích của ba số này
Tấ

là số chính phương.

• 2 6∈ S. Nếu S chứa một bộ trong các bộ (1; 4; 9), (3; 6; 8), (5; 12; 15) thì tích của ba
n

số đó là số chính phương. Xét trường hợp S chứa nhiều nhất hai số của mỗi bộ trên, khi
đó 7, 10, 11, 13, 14 ∈ S, do đó nếu 8 ∈ S thì 7 · 8 · 14 là số chính phương. Nếu 8 6∈ S thì
yễ

3, 6 ∈ S, khi đó nếu 12 ∈ S thì 1 · 3 · 12 là số chính phương, ngược lại 12 6∈ S thì 5, 15 ∈ S


nên 3 · 5 · 15 là số chính phương.
gu

Tóm lại trong S luôn chứa ba số có tích là số chính phương.


Vậy max |S| = 10.
N

Câu 1.8. Coi mỗi đấu thủ là một đỉnh của đồ thị. Hai đỉnh kề nhau nếu hai đấu thủ đã đấu
với nhau.  2
n
Nếu số trận đã đấu là không vượt quá + 1 thì ta xét phản ví dụ sau:
4  
hni n+1
Chia n đỉnh thành 2 nhóm, một nhóm đỉnh và một nhóm đỉnh. Nối mỗi đỉnh thuộc
2 2
nhóm 1 với tất cả các đỉnh thuộc nhóm 2, ngoài ra chọn 2 đỉnh tùy ý trong nhóm 1 và nối chúng
với nhau. Như vậy số cạnh là
hni n + 1  2
n
+1= +1
2 2 4

và rõ ràng không tồn tại 5 đỉnh thỏa mãn yêu cầubài toán.
n2
Ta chứng minh nếu số cạnh là không nhỏ hơn + 2 thì tồn tại 5 đỉnh thỏa mãn yêu cầu (1).
  4
25
Với n = 5: Số ván đã đấu là + 2 = 8. Nếu mỗi đỉnh có bậc lớn nhất là 3 thì số cạnh tối đa
4

28
1. CỰC TRỊ TRONG TỔ HỢP

5.3
là < 8 (vô lý). Do đó tồn tại ít nhất một đỉnh có bậc ít nhất là 4, giả sử là a. Gọi b,c,d,e là 4
2
đỉnh kề với a. Trong đồ thị con 4 đỉnh này luôn tồn tại 4 cạnh, suy ra phải tồn tại 2 cạnh không
chung đỉnh, giả sử là bc,de. Bài toán được khẳng định.  2
k
Giả sử (1) đã đúng đến n = k − 1 (k ≥ 6). Xét đồ thị k đỉnh với đúng + 2 cạnh.
  4
k
Nếu tồn tại 1 đỉnh có bậc không vượt quá thì bỏ đỉnh này đi và xét đồ thị con gồm k − 1
2
đỉnh còn lại. Khi đó tổng số cạnh của đồ thị con k − 1 đỉnh này ít nhất là
" #
(k − 1)2
 2          
k k k k+1 k k−1
+2− =2+ −1 =2+ =2+
4 2 2 2 2 2 4

Như vậy khẳng định đúng theo giả thiết quy nạp.
k
Nếu tất cả các đỉnh đều có bậc ít nhất là + 1.
2
Nếu k = 2l (l ≥ 3) thì tổng số cạnh ít nhất là
 2
2l (l + 1) 2 k
= l (l + 1) > l + 2 = + 2.

hu
2 4

Nếu k = 2l + 1 (l ≥ 3) thì tổng số cạnh ít nhất là

(2l + 1) (l + 1) T
≥ l (l + 1) + 2 =
 2
k
+ 2.
t
2 4
Tấ

Dấu bằng phải xảy ra nên l = 3,k = 7 và tất cả các đỉnh đều có bậc 4. 

Câu 1.9. Không mất tính tổng quát ta giả sử T = {1,2,3, · · · ,n} và gọi k(n) = max |X|.
n

Với 3 ≤ n ≤ 5 , khi đó hai tập con bất kì gồm ba phần tử của T luôn có giao khác rỗng. Do đó
k(n) = Cn3 .
yễ

Xét n ≥ 6, xét A là tập con gồm hai phần tử của tập {2,3, · · · ,n}. Khi đó các tập A ∪ {1} có 3
2
phần tử và hai tập bất kì có giao khác rỗng. Do vậy k(n) ≥ Cn−1 .
gu

Ta chứng minh
2
k (n) ≤ Cn−1 (1).
N

2
Giả sử k(m) ≤ Cm−1 với mọi m = 6,7, · · · ,n . Ta chứng minh (1) đúng với n + 1 .
Xét tập A ∈ X, không mất tính tổng quát ta giả sử A = {1,2,3}.
Nếu mọi B đều chứa hai phần tử của tập A thì |X| = 1 + 3 (n − 3) < Cn2 .
Giả sử có ít nhất một tập B chỉ chứa một phần tử của tập A, giả sử B = {1,4,5}. Nếu các tập
2
con của X đều chứa 1 thì |X| = Cn−1 .
Nếu có tập C ∈ X không chứa 1. Do A ∩ C 6= ∅, B ∩ C 6= ∅ nên ta giả sử C = {2,4,6}.
(n − 1) (n − 2) (n − 2) (n − 3)
Vì − = n − 2 nên nếu có không quá n − 2 tập chứa 7 thì do giả
2 2
thiết quy nạp ta có khẳng định đúng.
Trái lại giả sử có ít nhất n − 1 tập chứa số 7. Với mỗi tập như thế phải chứa 1,2 hoặc 3.
Nếu chứa 1 thì phải chứa 2,4 hoặc 6. Điều này xảy ra nếu tập đó là {1,2,7}, {1,4,7} hoặc {1,6,7}.
Nếu nó chứa 2 thì phải chứa 1,4 hoặc 5. Như vậy có thêm hai khả năng là {2,4,7} hoặc {2,5,7}.
Nếu nó chứa 3 thì nó phải chứa 1,4,5 nên chỉ có một khả năng {3,4,7}.
Như vậy có 6 khả năng cho tập chứa 7. Vì thế nếu n − 2 ≥ 6 thì có không quá n − 2 tập chứa 7.
Xét n = 7 , ta có T = {1,2, · · · ,7} ta lí luận tương tự và đi đến kết luận có không quá n − 2 tập
chứa 6. 
 0 nếu 1≤n<3
3
Vậy k(n) = C nếu 3≤n≤5 . 
 2n
Cn−1 nếu n≥6

29
1. CỰC TRỊ TRONG TỔ HỢP

Câu 1.10. Xét cách xếp 12 quân domino sau:

Ta thấy với cách xếp này thì ta không thể thêm một quân domino nào nữa. Ta chứng minh với
mọi cách xếp 11 quân đomino ta luôn có thể xếp thêm một quân domino nữa mà không cần xê
dịch các quân đomino. Giả sử ta không thể xếp thêm quân domino nào nữa. Khi đó hai ô kề
nhau thì luôn có một ô được phủ bởi quân domino.

hu
S2
T S3
t
Tấ

S1
n

Ta phân hoạch bảng 6 × 6 thành hai phần S1 , S2 trong đó S1 là bảng 5 × 6 và S2 gồm hàng cuối
yễ

cùng. Gọi A là tập các ô trống trong S2 . Vì trong S2 có nhiều nhất 3 ô trống nên trong S1 có ít
nhất 11 ô trống. Ta xét bảng S3 là bảng 5 × 6 bằng cách từ bảng 6 × 6 ta bỏ hàng trên cùng.
gu

Gọi B là tập các quân domino phủ trong S3 .


Xét v là một ô trống thuộc S2 , gọi t là ô nằm dưới ô v, khi đó t sẽ được phủ một quan domino
N

d nào đó.
Ta xét ánh xạ f : A → B như sau: f (v) = d. Ta chứng minh được f là đơn ánh, suy ra |A| ≤ |B|.
Mà |A| = 11 nên |B| ≥ 11. Do đó |B| = 11. Dẫn tới hàng trên cùng chỉ gồm các ô trống nên ta
có thể chèn thêm quân domino (vô lí).
Vậy nmin = 11. 

n a oĐặt A = {a1 ,a2 ,...,an } . Khi đó, trong A có phần tử ai lẻ (vì nếu các ai đều chẵn ta
Câu 1.11.
i
xét tập ).
2k
.
Suy ra S (A) − ai ..2 ⇒ S(A) lẻ.
.
Mà với mọi j 6= i thì S (A) − aj ..2 ⇒ aj lẻ.
n .
(S(A) − ak ) = (n − 1) S(A)..2 ⇒ n lẻ, dẫn đến n ≥ 5.
P
Suy ra
k=1
+) n = 5 ta có
A = {a1 < a2 < a3 < a4 < a5 } .
Bỏ a1 thì ta có a2 + a5 = a3 + a4 .
Bỏ a2 ta có a1 + a5 = a3 + a4 ⇒ a1 = a2 (vô lí).
+) n = 7 ta xét tập A = {1,3,5,7,9,11,13}. Ta thấy A thỏa mãn bài toán. 

30
1. CỰC TRỊ TRONG TỔ HỢP

Câu 1.12. Đặt Si = {x ∈ X : v3 (x) = i} ,i = 0, 1, 2, . . ., Tk = k; 3k,32 k, . . . với k = 1, 2, . . ..
S
Ta có Tk = X.
Bây giờ ta chọn số phần tử lớn nhất của A để trong A không có 2 số a, b để a = 3b.
Để có được điều đó, ta sẽ không chọn hai số nào liên tiếp nhau trong Tk .  
s s+1
Với s ≥ 0 lớn nhất thỏa 3 .k < 2013 thì trong Tk ta chọn được nhiều nhất số.
2
Ta chọn các số k, 32 k, 34 k, . . . . Hay ta chọn các phần tử thuộc S0 , S2 , S4 , S6 .
Mà |S0 | = 1342, |S2 | = 223, |S4 | = 24, |S6 | = 2.
Nên số phần tử tập A có thể chọn nhiều nhất để trong A không có 2 số a, b thỏa a = 3b là 1591.
Vậy nmin = 1592. 

Câu 1.13. Ta chia tập X thành các tập có một hoặc hai phần tử mà tổng của chúng là một
lũy thừa của 2 (nếu tập có một phần tử thì ta lấy 2 lần số đó).
Trước hết đếm số tập {a, b} sao cho a + b = 2048 = 211 , với a > b ta có a ≥ 1025 nên ta có các
tập
{2001; 47} , {2000; 48} , . . . , {1025; 1023} , {1024} .
Các tập ở trên gồm các số lớn hơn 46 nên ta còn lại những số nhỏ hơn 46. Ta đi xét tập {c; d}
mà c + d = 64, ta có các tập {46; 18} , {45; 19} , . . . , {33; 31} và {32}.

hu
Ta còn lại các số nhỏ hơn 18, khi đó tập {15; 17} và {16} có tổng hai phần từ bằng 32.
Các cặp số nhỏ hơn 15 có tổng bằng 16 là {14; 2} , {13; 3} , . . . , {9; 7}và {8}, {1} là tập cuối
cùng.
Ta có tất cả 998 cặp phân biệt có tổng là một lũy thừa của 2.
Xét tập A ⊂ X, |A| = 999.
T
t
+) Nếu trong A chứa một trong các số 1, 8, 16, 32, 1024 thì trong A luôn tồn tại hai số có tổng
Tấ

là lũy thừa của 2.


+) Nếu trong A không chứa các số 1, 8, 16, 32, 1024.
2001 − 5
Do = 998 mà |A| = 999 nên trong A luôn tồn tại hai số có tổng là lũy thừa của 2.
n

2
Xét các tập
yễ

A1 = {1025, 1026, . . . , 2001} , A2 = {33, 34, . . . , 46} , A3 = {17} , A4 = {9, 10, . . . , 14}
gu

và A = A1 ∪ A2 ∪ A3 ∪ A4 ta có |A| = 998 và trong A không có hai số nào có tổng là lũy thừa


của 2.
N

Vậy mmin = 999. 

Câu 1.14. Trước hết ta chứng minh trong S luôn có ít nhất một khoảng cách chia hết cho 3.
Kí hiệu các điểm đó là A, B, C, D và ta giả sử trong các khoảng cách AB, BC, CD, DA, AC, BD
không có khoảng cách nào chia hết cho 3.
Không mất tính tổng quát, ta giả sử BAD÷ > BAC ÷ > CAD.÷
Đặt α = BAC,
÷ β = CAD ÷ ⇒ BAD ÷ = α + β.

Áp dụng định lí Cô sin cho các tam giác BAC, CAD, BAD ta có:

BC 2 = AB 2 + AC 2 − 2AB.AC. cos α (1)


CD2 = AD2 + AC 2 − 2AC.AD. cos β (2)
BD2 = AB 2 + AD2 − 2AB.AD. cos (α + β) . (3)

Đặt x = 2AB.AC. cos α, y = 2AC.AD. cos β, z = 2AB.AD. cos (α + β).


Từ (1),(2), (3) và đề bài ta suy ra x, y, z là các số nguyên không chia hết cho 3. Suy ra

2AC 2 .z = 4AC 2 .AB.AD cos (α + β) = 4AC 2 .AB.AD (cos α cos β − sin α sin β)
= xy − 4AC 2 .AB.AD. sin α. sin β

31
1. CỰC TRỊ TRONG TỔ HỢP

là số nguyên.
Nên 4AC 2 .AB.AD. sin α. sin β là số nguyên chẵn không chia hết cho 3.
Do đó sin α. sin β là số hữu tỉ mà khi viết dưới dạng phân số tối giản thì tử số không chia hết
cho 3.
Đặt p = 2AB.AC, q = 2AC.AD ⇒ p, q không chia hết cho 3.
x y
Khi đó cos α = , cos β = nên
p q
p
p
2 2
(p2 − x2 ) (q 2 − y 2 )
sin α sin β = (1 − cos α) (1 − cos β) = .
pq
p
Ta có (p2 − x2 ) (q 2 − y 2 ) là số nguyên và không chia hết cho 3.
Nhưng p, q, x, y không chia hết cho 3 nên

p2 ≡ q 2 ≡ x2 ≡ y 2 ≡ 1 (mod3) .
p
Suy ra (p2 − x2 ) (q 2 − y 2 ) chia hết cho 3. Điều này dẫn đến vô lí.
Do đó trong các khoảng cách AB, BC, CD, DA, AC, BD có ít nhất một khoảng cách chia hết
cho 3.
Ta xét 4 điểm A, B, C, D như sau:

hu
Tam giác ABC cân tại A , D là trung điểm cạnh BC và AB = AC = 5, BC = 8.
Khi đó AD = 3, BD = CD = 4 nên trong các khoảng cách thì chỉ có một khoảng cách chia hết
cho 3.
Vậy trong S có ít nhất một khoảng cách chia hết cho 3. T 
t
Câu 1.15. Xét các tô 3 màu như sau
Tấ

Màu 1 : 1, 2, 6, 7, 9, 18, 20,


Màu 2 : 3, 4, 11, 12, 15, 16 và
n

Màu 3 : 5, 8, 10, 14, 17, 19.


yễ

Rõ ràng với cách tô trên thì không có ba số nào tô cùng màu lập thành cấp số cộng.
Ta chứng minh, nếu tô các số trên bởi 2 màu thì luôn có 3 số tô cùng màu lập thành cấp số cộng.
gu

Giả sử tồn tại một các tô các số bởi hai màu mà không có ba số cùng màu nào lập thành cấp số
cộng. Khi đó, ba số liên tiếp nhau sẽ được tô bởi hai màu khác nhau. Hơn nữa, với hai số a, b tô
N

cùng màu thì hai số 2a − b, 2b − a khác màu với chúng.


Xét ba số liên tiếp 9, 10, 11. Khi đó ta có các khả năng sau
1) Số 10 khác màu với số 9 và 11. Khi đó 2.9 − 11 = 7 và 2.11 − 9 = 13 tô cùng màu với 10.
Tuy nhiên ba số 7, 10, 13 lập thành cấp số cộng.

2) Số 10 cùng màu với 9 (tương tự như cùng màu với 11). Khi đó 2.9 − 10 = 8 tô cùng màu
với 11, do đó 2.11 − 8 = 14 và 2.8 − 11 = 5 tô cùng màu với 10, tương tự 2.5 − 9 = 1 và
2.10 − 14 = 6 tô cùng màu với 11. Tuy nhiên 1, 6, 11 lập thành cấp số cộng.
Vậy kmin = 3. 

Câu 1.16. Xét n tam giác thỏa yêu cầu bài toán (n ∈ N, n ≥ 3).
Xét một tam giác bất kì trong n tam giác này, kí hiệu là T0 . Theo đề bài thì T0 có điểm chung
với mỗi tam giác tỏng n − 1 tam giác còn lại, nên có môt đỉnh của T0 , kí hiệu A là đỉnh chung
n−1
của ít nhất k = , kí hiệu T1 , T2 , · · · , Tk .
3
Hơn nữa không có điểm nào là đỉnh chung của tất cả các tam giác nên có ít nhất một tam giác
T không nhận A làm đỉnh.
Theo giả thiết đề bài thì tam giác T có một điểm chung (khác A) với các tam giác T1 , T2 , · · · , Tk

32
1. CỰC TRỊ TRONG TỔ HỢP

và với i 6= j thì điểm chung của T với Ti khác điểm chung của T với Tj . Suy ra T có ít nhất k + 1
đỉnh, dẫn tới k + 1 ≤ 3, hay k ≤ 2. Từ đây ta có n ≤ 7.
Xét 7 tam giác ABC, ADE, AM N, BDM, BEN, CDN và CM E thỏa yêu cầu bài toán.
Vậy nmax = 7. 

Câu 1.17. Đặt A = {a1 , a2 , · · · , ak } với a1 < a2 < · · · < ak . (1)


Xét ba số a1 , a1 , ak ta có ( (
a1 + a1 > ak 2a1 > ak
2 2 2 ⇔ .
a1 + a1 ≥ ak 2a21 ≥ a2k
Từ (1) ta có ak ≥ a1 + k − 1, suy ra
√ 
2
2a21 ≥ (ak + k − 1) ⇔ a1 ≥ (k − 1) 2+1 .

√ 2018
Suy ra ak ≥ (k − 1)(2 + 2). Mà ak ≤ 2018 nên k ≤ √ + 1, hay k ≤ 592.
2+ 2
Xét tập A = {1427, 1428, · · · , 2018} ta có |A| = 592 và 14272 + 14272 > 20182 nên tập A thỏa
yêu cầu bài toán. 

hu
Câu 1.18. Rõ ràng mỗi cặp điểm đỏ thuộc tối đa hai đường tròn có tâm màu xanh. Gọi n là
n(n − 1)
số điểm đỏ thì số cặp tương ứng sẽ là và sẽ có không quá n(n − 1) đường tròn có tâm
2
xanh.
T
Suy ra tổng số điểm không vượt quá n(n − 1) + n = n2 điểm. Do đó n2 ≥ 2009 hay n ≥ 45.
Khi đó, số điểm xanh không vượt quá 2009 − 45 = 1964 điểm.
t
Ta sẽ chỉ ra cách xây dựng thỏa đề bài.
Tấ

Xếp 45 điểm đỏ phân biệt lên một đoạn thẳng có độ dài là 1. Sau đó vẽ 45 đường tròn đơn vị có
tâm đỏ. Rõ ràng chúng sẽ đôi một cắt nhau và các giao điểm đều phân biệt. Tổng số giao điểm
là 45 · 44 = 1980. Ta sẽ tô màu xanh cho đúng 1964 giao điểm trong số đó thì rõ ràng mỗi đường
n

tròn đơn vị có tâm là điểm xanh sẽ đi qua đúng hai điểm đỏ. Do đó, mô hình này thỏa mãn đề
yễ

bài.
Vậy số điểm xanh lớn nhất là 1964. 
gu

Câu 1.19. Ta phát biểu lại bài toán như sau Đặt X = {1, 2, · · · , n}. Biết tồn tại 6 tập con
A1 , A2 , · · · , A6 của X thỏa
N

i) |Ai | = 5, i = 1,6
ii) |Ai ∩ Aj | ≤ 2, i, j ∈ {1, 2, 3, · · · , 6} ; i 6= j.
Tìm min n.
Xét bảng gồm 6 hàng và n cột.
1 2 ········· n
A1 →5
A2 →5
··· ··· ··· ··· ··· →5
A6 →5
c1 c2 cn
n
P
Trong đó ci là số tập hợp Aj chứa phần tử i. Ta có ci = 30. Ta đếm số cặp số 1 trên cột:
i=1
Trên cột i có C2ci cặp số 1, do đó cặp số 1 trên bảng là
n n
X X c2 − ci
i
C2ci = .
i=1 i=1
2

33
1. CỰC TRỊ TRONG TỔ HỢP

Mặt khác với i 6= j thì trên hai hàng i, j có nhiều nhất 2 cặp số 1. Suy ra số cặp số 1 không vượt
quá 2.C26 = 30.
Do đó n n n
X X X
c2i − ci ≤ 60 ⇒ c2i ≤ 60 +

ci = 90.
i=1 i=1 i=1

Áp dụng bđt Cauchy – Schwarz ta có:


 n
2
P
n ci
X i=1 900
90 ≥ c2i ≥ = ⇒ n ≥ 10.
i=1
n n

Ta xây dựng trường hợp dấu “=” xảy ra


1 2 3 4 5 6 7 8 9 10
A1 × • • • • • • • • •
A2 × • • • • • • • • •
A3 × • • • • • • • • •
A4 • • • • • • • • • •

hu
A5 • • • • • • • • • •
A6 • • • • • • • • • •

Câu 1.20. Gọi Hi là thí sinh thứ i và tập các bài toán là {b1 ,b2 , . . . ,b9 }.
T 
t
Tấ

Theo đề bài ta có:


|Hi ∩ Hj | ≤ 1, ∀i 6= j.
Đặt ni là số thí sinh giải được bài bi .
n

Ta đi đếm bộ (bi ,Hj ,Hl ), trong đó bi ∈ Hj ∩ Hl .


Ta có số bộ này chính bằng:
yễ

X
|Hi ∩ Hj |.
gu

i<j

Mặt khác: số bộ này lại bằng


9
N

X
Cn2i .
i=1

Do đó ta có:
X 9
X
|Hi ∩ Hj | = Cn2i .
i<j i=1

Suy ra
9
X X
(n2i − ni ) ≤ 2 2
|Hi ∩ Hj | ≤ 2.C11 = 110 ⇒ 9(k 2 − k) ≤ 110 ⇒ k ≤ 4.
i=1 i<j

Với k = 4. Giả sử tồn tại ni ≥ 5, suy ra


9
X
(d2i − di ) ≥ 8.12 + 20 = 116 (vô lí).
i=1

Suy ra X
2
ni = 4,∀i = 1,9 ⇒ |Hi ∩ Hj | = 54 = C11 − 1.
i<j

34
1. CỰC TRỊ TRONG TỔ HỢP

Do đó, tồn tại (i; j) sao cho Hi ∩ Hj = ∅. Giả sử H1 ∩ H2 = ∅ và


|Hi ∩ Hj | = 1, ∀i < j,(i; j) 6= (1; 2.)
Nếu tồn tại i để |Hi | ≤ 3, ∀i 6= 1,2 thì suy ra
|Hi ∩ Ht | = 1, ∀t ∈ {1,2,...,11} \ {i} .
 
10
Nên tồn tại một phần tử của Hi thuộc ít nhất + 1 = 4 tập Ht ,t 6= i.
3
Suy ra tồn tại một phần tử thuộc nhiều hơn 5 tập Hj , vô lí.
Suy ra
X11
|Hi | ≥ 4 ⇒ |Hi | ≥ 36 + |H1 | + |H2 | > 36 ( vô lí.)
i=1

Do đó k ≤ 3. Với k = 3 ta chỉ ra như sau:


Quy ước : số 1 là thí sinh giải được bài đó; số 0 là thí sinh không giải được bài đó.
b1 b2 b3 b4 b5 b6 b7 b8 b9
H1 1 0 0 1 0 0 1 0 0
H2 1 0 0 0 1 0 0 0 1

hu
H3 0 1 1 0 0 1 0 1 0
H4 0 1 0 1 1 0 0 0 0
H5
H6
0
0
0
0
0
1
1
0
0
0 T
0
0
0
1
1
0
1
1
t
H7 0 0 0 0 1 1 0 0 0
Tấ

H8 1 1 0 0 0 0 0 0 0
H9 0 0 1 0 0 0 0 0 0
H10 0 0 0 0 0 1 1 0 0
H11 0 0 0 0 0 0 0 1 0
n
yễ

Câu 1.21. a) Giá trị nhỏ nhất của S là 0.


gu

−−−→
Ta xét cách xây dựng vectơ như sau: với hai đỉnh Ai ,Aj mà i > j, ta định hướng Ai Aj . Khi đó,
trong mỗi tam giác, luôn có một đỉnh có 2 vectơ hướng ra, như thế thì tổng các vectơ trên các


N

cạnh của chúng không thể là 0 được.


b) Ta gọi một góc là khác hướng nếu đỉnh của nó là một trong các đỉnh của đa giác đã cho, trên
hai cạnh, hai vectơ đã chọn có hướng đi ra và đi vào. Như thế ta thấy rằng:
- Tam giác thỏa mãn điều kiện đề bài, gọi là tam giác “đẹp”, có số góc khác hướng là 3.
- Tam giác không thỏa mãn có số góc khác hướng là 1.
3 3
Tổng số tam giác là C17 nên rõ ràng, có tổng cộng S · 3 + (C17 − S) · 1 = 680 + 2S góc khác hướng.
Tại một đỉnh Ai nào đó, gọi x,y lần lượt là số vectơ có gốc là Ai , có ngọn là Ai . Khi đó, số góc
2
khác hướng tại Ai là xy. Tuy nhiên, x + y = 16 nên xy ≤ (x+y) 4
= 64.
Do đó, tổng số góc khác hướng tại tất cả các đỉnh không vượt quá 17 · 64 = 1088. Suy ra
680 + 2S ≤ 1088 ⇔ S ≤ 204.
Để có mô hình thỏa mãn, ta thấy rằng tại mỗi đỉnh, số vectơ vào và ra phải bằng nhau và bằng
8. Ta xây dựng như sau: Với mỗi đỉnh i, ta có 2 trường hợp:
−−−→
* Nếu j < i và j cùng tính chẵn lẻ với i thì nối từ Ai Aj ; nếu j khác tính chẵn lẻ với i thì nối
−−−→
Aj Ai .
−−−→
* Nếu j > i và j cùng tính chẵn lẻ với i thì nối từ Aj Ai ; nếu j khác tính chẵn lẻ với i thì nối
−−−→
Ai Aj .
Dễ dàng kiểm tra được các xây dựng trên thỏa mãn ràng buộc của bài toán. 

35
1. CỰC TRỊ TRONG TỔ HỢP

Câu 1.22. Xét tập hợp S = {1010, 1012, · · · , 2016} ta thấy |S| = 504 và hai phần tử bất kì
thuộc S đều không nguyên tố cùng nhau và không có hai số nào thuộc S mà số này chia hết cho
số kia.
Xét A ⊂ X và |A| > 504 thỏa yêu cầu bài toán. Xét a = min x.
x∈A
Nếu 2a < 2017, ta xét tập A0 = ({2a} ∪ A) \{a}. Khi đó tập A0 thỏa yêu cầu bài toán. Thật vậy:
.
Với x, y ∈ A0 , x < y ta có y 6= 2x và 2x < 2y nên 2x 6 ..y. Hơn nữa (x,y) > 1 nên (2x,y) > 1. Do
đó A0 thỏa yêu cầu bài toán.
Từ đó ta chỉ cần xét A ⊂ {1009, 1010, · · · , 2016} (do 2017 là số nguyên tố nên 2017 6∈ A).
Vì hai số liên tiếp là nguyên tố cùng nhau nên |A| ≤ 504 (vô lí). 

Câu 1.23. Ta đếm số bộ (A, B, C) với hai cột A, B và hàng C giao nhau tại hai ô được tô
màu.
(1) Đếm theo hàng. Gọi xi là số ô được tô màu ở hàng thứ i.
p2 −p+1
Cx2i .
P
Số các cặp các ô được tô màu thuộc cùng một hàng là S =
i=1
(2) Đếm theo hai ô A, B.
Số cách chọn hai cột A,B là Cp22 +p+1 và có không quá một hàng C tương ứng cắt hai hàng tại
hai ô được tô màu (do giả thiết không có hình chữ nhật) nên S ≤ Cp22 +p+1 .

hu
Gọi α là số lượng lớn nhất các ô vuông có thể tô thì
p2 +p+1
X
xi = α. T
t
i=1
Tấ

Suy ra
p2 +p+1  
X 1 1
Cp22 +p+1 ≥ Cx2i ≥ 2
α2 − α
2 p +p+1
n

i=1

nên α ≤ (p + 1)(p2 + p + 1).


yễ

Ta sẽ chỉ ra một cách tô thỏa mãn đề bài.


Xét các bộ (a,b,c) mà 0 ≤ a,b,c ≤ p − 1 với a + b + c > 0 thì có tất cả p3 − 1 bộ.
gu

Ta xem 2 bộ (a,b,c) và (d,e,f ) thuộc cùng một lớp khi và chỉ khi ∃k ∈ {1,2,3,...,p − 1} sao cho
a ≡ kd,b ≡ ke,c ≡ kf ( mod p) nên có tất cả p2 + p + 1 lớp và ta đánh số chúng từ 1 đến p2 + p + 1.
N

Nếu có hai bộ (a,b,c) và (d,e,f ) lần lượt thuộc lớp i,j mà ad + be + cf chia hết cho p thì tô màu
ô (i,j).
Thật vậy, mỗi hàng có đúng p + 1 ô. Ta sẽ chứng minh rằng ax + by + cz ≡ 0(modp) có đúng
p + 1 nghiệm (2 nghiệm thuộc một lớp thì coi như là một nghiệm).
Có p2 − 1 bộ (x,y) mà ứng với chúng, tồn tại duy nhất z sao cho ax + by + cz chia hết cho p.
Mỗi lớp có p − 1 nghiệm nên có p + 1 nghiệm. Nhân số lượng này với số cột, ta có đpcm.
Tiếp theo, ta sẽ chứng minh rằng không tồn tại hình chữ nhật. Hệ sau có không quá một nghiệm
(
ax + by + cz ≡ 0(modp)
dx + ey + f z ≡ 0(modp)

b c
Giả sử 6= hay bf 6= ce thì với mọi x0 , tồn tại duy nhất y0 sao cho by + cz = α,ey + f z = β.
e f
Tuy nhiên, có p giá trị x0 và nghiệm (0,0,0) bị loại đi nên có không quá p − 1 nghiệm.
Chú ý rằng nếu (x,y,z) là nghiệm của hệ đồng dư trên thì (kx,ky,kz) cũng thỏa mãn, dẫn đến
có không quá 1 nghiệm. Từ đó ta có điều kiện đủ của bài toán.
Vậy GTLN cần tìm là α = (p + 1)(p2 + p + 1).


36
1. CỰC TRỊ TRONG TỔ HỢP

2
Câu 1.24. Ta có số đường chéo của đa giác đang xét là: C4n−1 − n = 2(n − 1)(4n − 1).
Ta chứng minh k ≤ (n − 1)(4n − 1).
Giả sử k > (n − 1)(4n − 1), khi đó sẽ tồn tại tập Si chỉ chứa một đường chéo, ta gọi đường chéo
đó là d. Gọi m là số đỉnh của đa giác nằm về một phía so với d. Khi đó, số đường chéo của đa
giác cắt d là m(4n − 3 − m) ≤ (2n − 1)(2n − 2).
Suy ra k ≤ m(4n − 3 − m) + 1 ≤ (2n − 1)(2n − 2) + 1 = (n − 1)(4n − 1) − (n − 2) ≤ (n − 1)(4n − 1)
vô lí.
Ta chỉ ra kmax = (n − 1)(4n − 1).
Với t = 2,3, · · · , n; i = 1,2, · · · , 4n − 1 và quy ước Ai+(4n−1) = Ai ta xét tập

Sti = {Ai Ai+t , Ai+t−1 Ai+2n }.

Dễ thấy có (n − 1)(4n − 1) tập Sti . Ta chứng minh với hai tập Sit và Si0 t0 bất kì luôn có hai đường
chéo nằm trong hai tập cắt nhau. 

n2
Câu 1.25. Số các cặp học sinh mà trong đó không ai gọi cho nhau không vượt quá .
4
Số các cặp học sinh mà trong đó có một người này gọi cho người kia, theo giả thiết không vượt
quá 3n.
n(n − 1) n2

hu
Do đó ≤ + 3n ⇔ n ≤ 14.
2 4
Giả sử 14 bạn là B1 , . . . , B7 và G1 , . . . , G7 . Khi đó theo vòng tròn Bi gọi cho Bi+1 , Bi+2 , Bi+3 và

Vậy giá trị lớn nhất có thể của n là 14. T


Gi gọi cho Gi+1 , Gi+2 , Gi+3 với i = 1, . . . , 7 (chỉ số lấy modulo 7)

t
Câu 1.26. Gọi k là giá trị lớn nhất có thể có. Khi đó, bằng phương pháp quy nạp, ta sẽ chứng
Tấ

tỏ rằng nếu S = {1; 2; . . . ; n} thì k = 2n − 3. Trước hết ta thấy rằng các tập

{1},{2}, . . . ,{n},{1; 2},{1; 2; 3}, . . . ,{1; 2; . . . ; n − 2}


n

gồm 2n − 3 các tập con của S thỏa mãn điều kiện bài toán.
yễ

Với n = 2, rõ ràng k = 1. Với n = 3 dễ dàng kiểm tra được rằng k ≤ 3 và các tập {1},{2},{3}
thỏa điều kiện đề bài, nên suy ra k = 3. Giả thiết quy nạp rằng kết quả đúng với n − 1 ≥ 3 (thì
gu

k = 2n − 5). Ta cần chứng minh kết quả cũng đúng với các tập S = {1; 2; . . . ; n}. Theo nhận
xét ban đầu suy ra k ≥ 2n − 3.
N

Gọi M = {A1 ; A2 ; . . . ; Ak } là tập có số phần tử nhiều nhất thỏa mãn điều kiện của bài toán.
Ta nhận xét rằng:
• Tập ∅ và S đều không thuộc M.

• Nếu tồn tại i sao cho 1 ≥ i ≥ n mà {i} hay {i}0 không thuộc M thì ta có thể thêm một
trong các số này vào M để mở rộng tập M.

• Cả hai tập {i} và {i}0 không thể cùng thuộc tập M, do đó chỉ có đúng {i} hoặc {i}0 thuộc
n
M. Vì vậy ta có thể giả sử |Ai | ≤ với mọi 1 ≤ i ≤ n.
2
Do 2n − 3 > n, nên trong M có một tập có số phần tử lớn hơn hoặc bằng 2, vì thế ta có thể
chọn một tập A ∈ M sao cho |A| ≥ 2 và |A| ≤ |X| với mọi X ∈ M mà |X| ≥ 2. Không mất tính
tổng quát ta giả sử 1,2 ∈ A. Khi đó xét bất kỳ tập B ∈ M khác {1},{2} và A.
• Nếu A ∩ B = ∅ thì 1,2 ∈ B.

• Nếu A ∩ B 0 = ∅ thì A ⊂ B.

• Nếu A0 ∩ B = ∅ thì B ⊂ A và khi đó |B| = 1, theo cách chọn A và lấy B ∈ M, suy ra


1,2 ∈
/ B.

37
1. CỰC TRỊ TRONG TỔ HỢP

n−1
• Nếu A0 ∩ B 0 = ∅ thì A ∪ B = S. Khi đó, nếu n là số lẻ thì do |A|,|B| ≤ ta có
2
|A ∪ B| ≤ n − 1 (vô lí); khi n là số chẵn thì chỉ có một trường hợp có thể xảy ra đó là
n
|A| = |B| = , nhưng khi đó B = A0 và A ∩ B = ∅, suy ra 1,2 ∈ / B.
2
Do vậy ta kết luận {1; 2} ⊂ B hoặc {1; 2} ∩ B = ∅ với mọi B ∈ M, khác {1},{2} và A. Khi
đó, bằng cách loại bỏ {1} và {2} từ M và loại bỏ 1 từ mỗi phần tử của M ta nhận được một
tập M mới ứng với n − 1 phần tử của tập hợp S = {2; 3; . . . ; n}. Theo giả thiết quy nạp ta có
k − 2 ≤ 2n − 5 mà k ≥ 2n − 5 nên ta được k = 2n − 3. Vậy khi n = 2017 thì giá trị lớn nhất cần
tìm có thể có của k là k = 2 · 2017 − 3 = 4031.


Câu 1.27. Gọi bảng ô vuông là ABCD. Tô màu đen trắng cho các ô vuông xen kẽ theo kiểu
bàn cờ sao cho các ô ở góc A, C màu trắng và các ô ở góc B, D màu đen. Do mỗi đường đi chéo
là một tập hợp các ô vuông cùng màu, nên ta chỉ cần tìm số đường đi chéo rời nhau ít nhất mà
các tập ô vuông màu trắng có thể phân hoạch thành. Ta điền vào mỗi ô trắng số 1 hoặc số 0 như
hình vẽ.

hu
A B
1 1 ••• 1
0 ••• 0 T 1
t
1 1 ••• 0
Tấ

• • • • • •
• • • • • •
n

• • • • • •
yễ

0 ••• 1 1
gu

1 0 ••• 0
•••
N

1 1 1
D C

Theo đó, màu sắc và số điền trên các ô đối xứng nhau qua đường chéo BD. Đồng thời trên mỗi
phía của BD, mỗi cặp ô kề nhau thì được điền số khác nhau.
Giả sử tập ô trắng được phân hoạch thành m đường đi chéo rời nhau. Khi một đường đi chéo có
hai ô kề nhau cùng điền số 1 thì ta tách đường đi đó làm đôi tại vị trí điểm chung của hai ô đó.
Ta tiếp tục làm như vậy khi còn có thể. Do trên bảng có đúng n cặp ô số 1 kề nhau cùng điền
một số nên số lần tách nhiều nhất là n, vì vậy số đường đi chéo tạo thành nhiều nhất là m + n.
Trên mỗi đường đi chéo bây giờ, hai ô kề nhau luôn có ít nhất một số 0, nên số ô điền 1 nhiều
hơn số ô điền 0 nhiều nhất là một. Mặt khác dễ thấy tổng số ô điền 1 nhiều hơn tổng số ô điền 0
ở trên bảng là 2n. Do đó số đường đi chéo lúc này ít nhất là 2n. Vì vậy m + n ≥ 2n, hay m ≥ n.
Kết hợp với tập các ô đen, ta có số đường đi chéo ít nhất là 2n.
Cuối cùng ta chỉ ra một cách phân hoạch thỏa mãn: Nối các ô đen của hai hàng đầu và nối các
ô trắng của hai hàng đầu lại, khi đó hai hàng đầu phân hoạch thành hai đường đi chéo. Do đó
ta có thể phân hoạch 2n hàng thành 2n đường đi chéo. 

Câu 1.28.

38
1. CỰC TRỊ TRONG TỔ HỢP

Trước tiên, ta chứng minh rằng nếu n ≤ 24 thì trong mọi trường hợp, yêu cầu của bài toán
đều không được thỏa mãn.
Xét I := {1; 2; 3; 4}. Mỗi bài làm của một học sinh có thể được đặt tương ứng với một bộ
x = (x1 ; x2 ; x3 ; x4 ; x5 ) ∈ I 5 ; trong đó, xi là thứ tự của phương án mà học sinh đã chọn để trả lời
cho câu hỏi thứ i (i ∈ Z, 1 ≤ i ≤ 5).
Ngoài ra, nếu x ∈ I 5 ta cũng chấp nhận cách viết x ≡ (x1 ; x0 ) với x0 := (x2 ; x3 ; x4 ; x5 ) ∈ I 4 . Bằng
cách như vậy, dễ thấy I 5 được phân hoạch thành 44 = 256 tập con (rời nhau); mỗi tập con gồm
đúng 4 bộ chỉ khác nhau ở thành phần thứ nhất, tức là tập con có dạng

Ax := {(1; x0 ), (2; x0 ), (3; x0 ), (4; x0 )} ⊂ I 5 , với x0 ∈ I 4 .

Vì 2000 > 7 × 256 nên theo nguyên tắc Dirichlet có 8 học sinh (khác nhau) A1 ,A2 ,...,A8 , mà bài
làm của họ ứng với các bộ thuộc cùng một tập con A nào đó (trong số 256 tập con nói trên).
Nhưng, 2000 − 8 = 1992 > 7 × 256 nên theo nguyên tắc Dirichlet lại có 8 học sinh B1 ,B2 ,...,B8
(khác nhau, trong số 1992 học sinh còn lại), có bài làm ứng với các bộ thuộc cùng một tập con
B nào đó (trong số 256 tập con nói trên).
Cuối cùng, vẫn có 1992 − 8 = 1984 > 7 × 256 nên dùng nguyên tắc Dirichlet một lần nữa ta có
8 học sinh C1 ,C2 ,...,C8 (khác nhau, trong số 1984 học sinh còn lại), có bài làm ứng với các bộ
thuộc cùng một tập con C nào đó (trong số 256 tập con nói trên). Từ 4 học sinh bất kỳ trong số

hu
24 học sinh A1 ,A2 ,...,A8 , B1 ,B2 ,...,B8 , C1 ,C2 ,...,C8 ta luôn chọn ra được hai học sinh có bài làm
ứng với các bộ cùng thuộc một tập con (một trong 3 tập A, B hoặc C); giả sử, chẳng hạn, đó là
các học sinh Ai , Aj (i,j ∈ Z; 1 ≤ i < j ≤ 8) mà bài làm ứng với hai bộ trong A. Theo cách xây
T
dựng các tập con ở trên, bài làm của hai học sinh này chỉ có thể khác nhau ở tối đa một câu hỏi
(chính là câu hỏi thứ nhất, nếu hai bài làm không hoàn toàn giống nhau); yêu cầu của bài toán
t
đã không được thỏa mãn.
Tấ

Bây giờ, ta chỉ ra tình huống mà n = 25 thỏa mãn  yêu cầu của bài toán.
5
Muốn vậy, xét S := x ∈ I 5 x1 ≡ 0 (mod 4) . Rõ ràng, S gồm đúng 4 × 4 × 4 × 4 × 1 = 256
P
i=1
n

bộ; hơn nữa, khi x,y ∈ S thì x 6= y ⇒ ∃i,j ∈ N∗ , i < j ≤ 5, xi 6= yi , xj 6= yj


Lấy D là một tập con 250 phần tử của S và xét tình huống mà: với mỗi x ∈ D tồn tại đúng 8
yễ

học sinh mà có bài làm ứng với bộ x(tình huống này hoàn toàn có thể xảy ra vì 2000 = 8 × 250).
Khi ấy, do 25 > 3 × 8, nên theo nguyên tắc Dirichlet, cứ 25 học sinh thì tìm được bốn học sinh
gu

(trong số 25 học sinh này) có bài làm ứng với bốn bộ x,y,z,t ∈ D ⊂ S đôi một khác nhau và khi
đó hai học sinh nào trong số bốn học sinh đó cũng có bài làm khác nhau ở ít nhất là hai câu hỏi,
N

đúng theo yêu cầu của bài toán. Vậy số tự nhiên bé nhất phải tìm là n = 25. 

Câu 1.29. Câu trả lời là: 2 nếu n − 2; 3 nếu n = 3; và n − 1 nếu n ≥ 4.


Với n = 2 thì rõ ràng 2 đội không hòa nhau, và do đó khoảng cách điểm duy nhất là 2.
Với n = 3 thì rõ ràng đội đứng đầu phải thắng ít nhất 1 trận. Nếu đội đứng đầu thắng 2 trận
thì đội này có 6 điểm. Trong trường hợp này, hai đội còn lại có 0 điểm. Khoảng cách giữa các
đội đứng đầu và đứng cuối trong trường hợp này là 6 điểm. Giả sử đội đứng đầu thắng 1 hòa 1.
Nếu 2 đội còn lại hòa nhau thì rõ ràng tổng số điểm của các đội phải là 4,2,1; do đó khoảng cách
giữa các đội đứng đầu và đứng cuối bảng là 3. Nếu 2 đội còn lại không hòa nhau, thì dễ thấy đội
cuối bảng phải là đội thua cuộc. Trong trường hợp này, 2 đội đầu bảng hòa nhau và có cùng số
điểm, mâu thuẫn. Vậy, với n = 3 thì khoảng cách tổi thiểu giữa đội đứng đầu và đứng cuối là 3.
Với n ≥ 4. Hiển nhiên rằng do n đội có số điểm đôi một khác nhau và là các số nguyên, khoảng
cách giữa đội đứng đầu và đội đứng cuối ≥ n − 1. Ta sẽ chỉ ra rằng nếu n ≥ 4 thì đánh giá này
hiển nhiên này là chặt. Cụ thể hơn, ta sẽ chứng minh bằng qui nạp khẳng định sau: với mọi
n ≥ 4, tồn tại một giải đấu với n đội sao cho sau khi giải đấu kết thúc, tổng số điểm của các đội
lần lượt là 2n − 3,2n − 4, . . . ,n − 2.
Ta lập luận bằng quy nạp. Với n ≥ 4 ta dễ dàng xây dựng được một giải đấu với khoảng cách
đội đầu và đội cuối là 3. Chẳng hạn với 4 đội A,B,C,D mà A thắng B, hòa với C,D; B hòa với
C và thua D; C hòa với D thì tổng số điểm các đội tương ứng là 5,4,3,2.

39
1. CỰC TRỊ TRONG TỔ HỢP

Giả sử tồn tại một giải đấu với n đội với tổng số điểm lân lượt là 2n − 3,2n − 4,n, . . . ,n − 2. Để
xây dựng một giải đấu với n + 1 đội, ta bổ sung đội thứ n + 1 vào giải đấu đó. Ta để đội n + 1
thắng đội thứ 1 (đội có 2n − 3 điểm), thua các đội thứ 2,3;thắng đội thứ 4, thua đội thứ 5,6,...
cho đến khi đội thứ n + 1 còn phải gặp 1,2,3 đội cuối cùng (tùy theo n ≡ 1,2 hay 0(mod3)).
Trong trường hợp thứ nhất (n ≡ 1(mod3)), ta để đội thứ n + 1 hòa đội thứ n; trong trường hợp
số hai, ta để đội thứ n + 1 hòa đội thứ n − 1 và thua đội thứ n; trong trường hợp thứ ba, ta để
đội thứ n + 1 hòa với đọi thứ n − 2, thua đội thứ n − 1 và thắng đội thứ n. Bằng các tính toán
đơn giản, ta dễ dàng kiểm tra được rằng trong cả ba trường hợp ta thu được giải đấu với n + 1
đội phân bố các điểm là 2n − 1,2n − 2, . . . ,n − 1. Bài toán được chứng minh.


Câu 1.30.

a) Xét bài toán tổng quát với bảng vuông với kích thước n × n(n ≥ 2).
Giả sử ngược lại, hai ô lân cận bất kì đều có hiệu nhỏ hơn hay bằng n − 1.
Với mỗi i = 1,2,...,n2 − n, ta đặt

Ai = {1,2,...,i},Bi = {i + 1,...,i + n − 1}, Ci = {i + n,...,n2 },

hu
ta nhận thấy trong số bất kì i ô vuông chứa các số thuộc Ai sẽ không kề với các ô chứa các
số thuộc Ci . Vì |B| = n − 1 < n nên với mọi i cố định, tồn tại dòng và cột được ghi các số
hoặc thuộc Ai hoặc thuộc Ci .
T
Với i = 1 thì tồn tại dòng và cột được ghi các số thuộc Ci , với i = n2 − n, tồn tại dòng,
cột được ghi các số thuộc An2 −n .
t
Giao các dòng và cột trên thì được ít nhất 2 ô được ghi các số ở cùng Am và Cm−1 . Điều
Tấ

này mâu thuẫn do Am ∪ Cm−1 = ∅.


Vậy luôn tồn tại hai số được điền vào hai ô lân cận có hiệu lớn hơn hay bằng n = 2017.
n

b) Theo câu a thì min k ≥ n = 2017.


Xét cách đánh số vào các ô (i,j) bất kì các số ai,j = (i − 1)n + j,∀i,j ∈ {1,2,...,n} với ai,j chỉ
yễ

số nằm ở hàng i (từ trên xuống), cột j (từ trái sang) thì mỗi số trong các số 1,2,...,20172
được điền vào các ô vuông của bảng đúng một lần
gu

Với ô (i,j) bất kì thì các ô lân cận là (i − 1,j); (i + 1,j),(i,j + 1),(i,j − 1) nên các số được
điền vào ô lân cận tương ứng là ai−1,j ,ai+1,j ,i,j − 1,i,j + 1 có hiệu không quá 2017.
N

Vậy min k = n = 2017.

Câu 1.31. Đầu tiên, ta sẽ mô tả tường minh về S để dễ cho việc đánh giá.
Theo định nghĩa về khoảng cách thì nó sẽ nhận các số trong tập hợp {1, 2, 3, 4, 5}. Do đó,
S = x1 + 2x2 + 3x3 + 4x4 + 5x5 trong đó xi , i = 1,5 là số cặp hai số đứng cạnh nhau trong H mà
có khoảng cách bằng i.
Bây giờ ta sẽ tìm cách đánh giá các xi , i = 1,5.

a) x5 ≥ 9 vì phải xuất hiện hết: . . . 0, . . . , . . . 9.

b) x4 + x5 ≥ 99 vì tối thiểu phải có . . . 00 → . . . 99.

c) x3 + x4 + x5 ≥ 999 vì tối thiểu phải có . . . 000 → . . . 999.

d) x2 + x3 + x4 + x5 ≥ 9999 vì tối thiểu phải có . . . 0000 → . . . 9999.

e) x1 + x2 + x3 + x4 + x5 ≥ 9 · 104 (số các số có 5 chữ số).

40
1. CỰC TRỊ TRONG TỔ HỢP

Cộng theo vế, ta có:

S = x1 + 2x2 + 3x3 + 4x4 + 5x5 ≥ 11106 + 9 · 104

Cuối cùng, ta chỉ cần xây dựng hoán vị để xảy ra dấu bằng.
Do ta cần chữ số khác nhau trong hai số xa nhất, tức là ta cần so sánh các chữ số bằng nhau ở
hai số từ phải qua trái. Do đó, nếu ta viết ngược lại các chữ số thì ta sẽ so sánh được sự lớn bé
của hai số với nhau. Như vậy hoán vị H là hãy xếp các số từ bé đến lớn: 00001,00002, . . . ,9999.
Sau khi đổi ngược lại thì nếu số nào không đủ 5 chữ số thì ta bỏ đi. Đó là hoán vị cần tìm. 

Câu 1.32.
a) Sau mỗi thao tác, tổng các số ở 4 góc là không đổi (1), tổng các số trong cả 9 ô giảm
nghiêm ngặt (2) và số nằm ở hình vuông chính giữa không tăng (3).
Trường hợp a), tổng 4 số ô góc là 20 nên nếu cuối cùng tất cả các ô bằng nhau thì tất cả
các ô đều chứa số 5. Tuy nhiên khi đó tổng 9 số trong 9 ô là 45, trong khi tổng ban đầu
cũng là 45. Mâu thuẫn với (2).
7
Trường hợp b), tổng 4 số 4 góc là 14, tức là nếu các số bằng nhau, mỗi ô đều điền . Mâu
2
7
thuẫn với (3) vì ban đầu ô ở giữa bằng 3, sau là , tức là sẽ tăng.

hu
2
Vậy, không thể đưa về bảng bằng nhau nếu bảng ban đầu như đề bài cho.

b) Theo nhận xét (2) thì M < 5. Nếu 4 < M < 5 thì M ≥
T 17
4
.
Gọi a là tổng độ tăng của các ô ban đầu chứa 9, 8, 7, 6, 5 thì tổng độ giảm của các ô này là
t
Tấ

(9 − x) + (8 − x) + (7 − x) + (6 − x) + (5 − x) + a = 35 − 5x + a.

Gọi b là tổng độ giảm của các ô ban đầu là 1, 2, 3, 4 thì tổng độ tăng trong các ô đó là
n

(1 + x) + (2 + x) + (3 + x) + (4 + x) + b = 4x + b − 10.
yễ

Do tổng số tăng bằng 1 nửa tổng số giảm nên 35 − 5x + a + b = 2(a + (4x + b − 10)) hay
55 − (a + b) 55
gu

x= suy ra x ≤ . Mâu thuẫn.


13 13
Vậy M ≤ 4. Sau đây là 1 ví dụ cho kết quả là M = 4.
N

9 7 2 7 5 4 3.5 4 3 4 4 4
8 5 6 −→ 8 5 6 −→ 4.5 4 5 −→ 4 4 4
1 3 4 1 3 4 4.5 4 5 4 4 4

Câu 1.33. Gọi các số viết trên đỉnh của hình lập phương là a, b, c, d, A, B, C, D. Khi đó

S = (b + d + A + C)(a + c + B + D) − T

với T = aA + bC + cC + dD.
Ta sẽ chứng minh T ≥ 12 82 + 22 72 + 32 62 + 42 52 .
Vì tập các giá trị của T là hữu hạn nên nó đạt được giá trị nhỏ nhất là T0 . Với một cách viết
nào đó ta có
T0 = a0 A0 + b0 B0 + c0 C0 + d0 D0 .
Không mất tính tổng quát, giả sử a0 = 82 . Ta thấy A0 = 1, thật vậy nếu A0 > 1, ta giả sử
B0 = 1, khi đó

T0 = 82 A0 + b0 + c0 C0 + d0 D0 > 82 + A0 b0 + c0 C0 + d0 D0 (vô lí).

41
1. CỰC TRỊ TRONG TỔ HỢP

Lập luận tương tự ta thu được T0 = 12 82 + 22 72 + 32 62 + 42 52 . Suy ra T ≥ T0 với mọi cách viết
số.
Theo bất đẳng thức AM-GM, ta có
 2
a+b+c+d+A+B+C +D
S≤ − T = 1022 − T
2
≤ 1022 − T0 ≤ 9420.

Mặt khác khi a = 12 , b = 32 , c = 72 , d = 52 , A = 82 , B = 62 , C = 22 , D = 42 , ta có S = 9420.


Vậy giá trị lớn nhất của S là 9420. 

Câu 1.34. Đáp số của bài toán là 1009


Gọi a là số câu hỏi mà A đã hỏi để có thể chắc chắn biết được tất cả các số ghi trên từng thẻ.
Ta có các nhận xét sau

◦ Trong một nhóm 3 thẻ đã được hỏi ở một lần hỏi nào đó có tối đa 1 thẻ không được đề
cập đến trong các câu hỏi khác. Thực vậy, nếu có ít nhất hai thẻ trong một câu hỏi nào đó
mà không được đề cập đến trong các thẻ khác thì ta chỉ biết được tập hai số ghi trên hai
thẻ ấy và không thể chắc chắn biết được số nào ghi trên thẻ nào.

hu
◦ Mỗi thẻ phải được đề cập trong ít nhất một câu hỏi nào đó. Thực vậy, nếu một thẻ nào đó
không được đề cập đến thì cho dù bạn A có biết tất cả các số ở các thẻ còn lại thì A cũng

T
không thể suy ra được số ghi trên thẻ kia, do A không biết được tập tất cả các số ghi trên
thẻ.
t
Tấ

Như vậy, chỉ có hai loại thẻ: Loại I gồm các thẻ chỉ xuất hiện một lần trong câu hỏi và Loại II
gồm các thẻ được xuất hiện hai lần trong các câu hỏi. Kí hiệu số thẻ Loại I là x thì số thẻ Loại
II là 2017 − x, trong đó 1 ≤ x ≤ 2017. Số lượt thẻ được nhắc trong toàn bộ các câu hỏi là 3a, do
đó ta có
n

3a ≤ x + 2 × (2017 − x) = 4030 − x
yễ

4034
Lại do nhận xét thứ hai nên x ≤ a. Thành thử 3a ≥ 4034 − a hay a ≥ = 1008.5 Vậy
4
gu

 
4034
a≤ = 1009
4
N

Thành thử A ít nhất cũng phải hỏi a0 = 1009 câu hỏi nếu muốn biết tất cả các số ghi trên từng
thẻ.
Ta chứng minh có một quá trình hỏi a0 câu hỏi là biết tất cả các số ghi trên từng thẻ. Xét một
nhóm 6 thẻ bất kì, tạm đánh số các thẻ ấy là 1,2,3,4,5,6. Bạn A sẽ hỏi Thầy giáo nhóm các thẻ
{1,2,3}, {3,4,5} và {5,6,1}. Sau khi nghe xong câu trả lời A sẽ suy ra được số ghi trên các thẻ
3,5,1 vì chúng là các thẻ xuất hiện đúng hai lần trong ba câu hỏi trên. Khi đó trong mỗi tập hợp
số ba thẻ đã hỏi, A đã biết tập hợp đó cũng như biết được hai số ghi trên hai thẻ tương ứng.
Từ đó A sẽ suy ra được số còn lại ghi trên hai thẻ nào. Ví dụ như trong nhóm {1,2,3} bạn A đã
biết được thẻ 1,3 ghi số gì và biết được tập hợp số ghi trên ba thẻ đó. Do đó A sẽ suy ra được
thẻ 2 ghi số gì ( do các số ghi trên các thẻ là đôi một phân biệt)
Vì 2017 ≡ ( mod 6) nên ta có thể chia 2017 thẻ đã cho thành các nhóm rời nhau, mỗi nhóm
2016 2016
6 thẻ và một thẻ dư ra. Theo lập luận trên A có thể hỏi Thầy ×3 = = 1008 câu
6 2
hỏi để biết hết tất cả số ghi trên từng thẻ. Cộng thêm với thẻ cuối cùng dôi ra, A chỉ cần hỏi
1008 + 1 = 1009 câu hỏi là sẽ rõ tất cả các số ghi trên từng thẻ. 

Câu 1.35. Với hai điểm Ai , Aj ta kí hiệu Ai Aj là cung bắt đầu từ Ai và kết thúc là Aj theo
chiều kim đồng hồ và kí hiệu m (Ai Aj ) là số đo của cung đó. Một cung được gọi là tù nếu

42
1. CỰC TRỊ TRONG TỔ HỢP

m (Ai Aj ) ≥ 180◦ .
Nhận thấy m (Ai Aj ) + m (Aj Ai ) = 360◦ nên có ít nhất 1 trong hai cung này tù.
Kí hiệu xs là số cung tù mà giữa hai đầu mút có đúng s − 1 điểm.
n
Nếu s 6= thì mỗi i có ít nhất một cung Ai Ai+s , Ai+s Ai là tù, tổng theo i ta được
2
xs + xn−s ≥ n.

Đẳng thức trên xảy ra khi không có đường kính Ai Ai+s Nhận thấy số tam giác không nhọn (tù
hoặc vuông) bằng số góc không nhọn.
Mỗi cung tù chứa s − 1 điểm thì có n − s − 1 tam giác không nhọn (dùng hai điểm đầu mút của
cung kết hợp với 1 điểm ngoài cung).
Số lượng các tam giác không nhọn là

N = x1 (n − 2) + x2 (n − 3) + · · · + xn−1 .2 + xn−2 .

Theo bất đẳng thức trên ta đánh giá được:


n−1
2
X 
n−3

n (n − 1) (n − 3)
N≥ (s − 1) (xn−s + xs ) ≥ n 1 + 2 + · · · + = nếu n lẻ.

hu
s=1
2 8


n−1
2 n−2
 Tn−4

n−2 n n (n − 2)2
t
X
N≥ (s − 1) (xn−s + xs )+ xn ≥ n 1 + 2 + · · · + + . = nếu n chẵn.
Tấ

s=1
2 2 2 2 8
2
Dấu bằng xảy ra ở các BĐT trên là không tồn tại 2 điểm đối xứng nhau qua tâm đường tròn.
n (n − 1) (n − 2)
n

Số lượng các tam giác có đỉnh là 3 trong các điểm trên là Cn3 = .
6
yễ

Vậy số tam giác nhọn là


gu

n (n − 1) (n − 2) n (n − 1) (n − 3) (n − 1) n (n + 1)
− = nếu n lẻ
6 8 24
n (n − 1) (n − 2) n(n − 2)2 (n − 2) n (n + 2)
N

− = nếu n chẵn.
6 8 24


Câu 1.36.
a) Xét bài toán tổng quát với bảng vuông với kích thước n × n(n ≥ 2).
Giả sử ngược lại, hai ô lân cận bất kì đều có hiệu nhỏ hơn hay bằng n − 1.
Với mỗi i = 1,2,...,n2 − n, ta đặt

Ai = {1,2,...,i},Bi = {i + 1,...,i + n − 1}, Ci = {i + n,...,n2 },

ta nhận thấy trong số bất kì i ô vuông chứa các số thuộc Ai sẽ không kề với các ô chứa các
số thuộc Ci . Vì |B| = n − 1 < n nên với mọi i cố định, tồn tại dòng và cột được ghi các số
hoặc thuộc Ai hoặc thuộc Ci .
Với i = 1 thì tồn tại dòng và cột được ghi các số thuộc Ci , với i = n2 − n, tồn tại dòng,
cột được ghi các số thuộc An2 −n .
Giao các dòng và cột trên thì được ít nhất 2 ô được ghi các số ở cùng Am và Cm−1 . Điều
này mâu thuẫn do Am ∪ Cm−1 = ∅.
Vậy luôn tồn tại hai số được điền vào hai ô lân cận có hiệu lớn hơn hay bằng n = 2017.

43
1. CỰC TRỊ TRONG TỔ HỢP

b) Theo câu a thì min k ≥ n = 2017.


Xét cách đánh số vào các ô (i,j) bất kì các số ai,j = (i − 1)n + j,∀i,j ∈ {1,2,...,n} với ai,j chỉ
số nằm ở hàng i (từ trên xuống), cột j (từ trái sang) thì mỗi số trong các số 1,2,...,20172
được điền vào các ô vuông của bảng đúng một lần
Với ô (i,j) bất kì thì các ô lân cận là (i − 1,j); (i + 1,j),(i,j + 1),(i,j − 1) nên các số được
điền vào ô lân cận tương ứng là ai−1,j ,ai+1,j ,i,j − 1,i,j + 1 có hiệu không quá 2017.
Vậy min k = n = 2017.

Câu 1.37. Gọi các số viết trên đỉnh của hình lập phương là a, b, c, d, A, B, C, D. Khi đó

S = (b + d + A + C)(a + c + B + D) − T

với T = aA + bC + cC + dD.
Ta sẽ chứng minh T ≥ 12 82 + 22 72 + 32 62 + 42 52 .
Vì tập các giá trị của T là hữu hạn nên nó đạt được giá trị nhỏ nhất là T0 . Với một cách viết
nào đó ta có
T0 = a0 A0 + b0 B0 + c0 C0 + d0 D0 .

hu
Không mất tính tổng quát, giả sử a0 = 82 . Ta thấy A0 = 1, thật vậy nếu A0 > 1, ta giả sử
B0 = 1, khi đó

T
T0 = 82 A0 + b0 + c0 C0 + d0 D0 > 82 + A0 b0 + c0 C0 + d0 D0 (vô lí).
t
Lập luận tương tự ta thu được T0 = 12 82 + 22 72 + 32 62 + 42 52 . Suy ra T ≥ T0 với mọi cách viết
Tấ

số.
Theo bất đẳng thức AM-GM, ta có
 2
a+b+c+d+A+B+C +D
n

S≤ − T = 1022 − T
2
yễ

≤ 1022 − T0 ≤ 9420.
gu

Mặt khác khi a = 12 , b = 32 , c = 72 , d = 52 , A = 82 , B = 62 , C = 22 , D = 42 , ta có S = 9420.


Vậy giá trị lớn nhất của S là 9420. 
N

Câu 1.38. Đặt n = ak + r với a ∈ Z+ và 0 ≤ r < k. Ta sẽ chứng minh 2 nhận xét sau:
Nhận xét. Nếu gọi T,S lần lượt là tổng các số được điền vào bảng ô vuông có kích thước r × r
và (k − r) × (k − r) nào đó thì

|T | ≤ t = min r2 ,k 2 − r2 và |S| ≤ s = min (k − r)2 ,k 2 − (k − r)2 .


 

Rõ ràng mỗi bảng ô vuông kích thước r × r đều bị chứa trong một bảng ô vuông kích thước k × k
nào đó mà tổng các số bằng 0. Gọi T 0 là tổng các số còn lại trên bảng k × k sau khi loại bảng
r × r chứa trong đó. Ta có
T + T 0 = 0 ⇒ |T | = |T 0 | .
Số lượng các ô trên bảng r × r là r2 nên |T | ≤ r2 và số lượng các ô còn lại của bảng k × k là
k 2 − r2 nên |T 0 | ≤ k 2 − r2 . Do đó:

|T | ≤ t = min r2 ,k 2 − r2


Hoàn toàn tương tự với bảng (k − r) × (k − r).


Nhận xét. Nếu kích thước của bảng tăng lên k đơn vị thì tổng các số trên bảng sẽ tăng không
quá t + s.

44
1. CỰC TRỊ TRONG TỔ HỢP

Xét hình minh họa sau (với k = 3,r = 2) , trong đó từ một bảng ban đầu, ta thêm vào phía trên
và bên phải tương ứng k hàng và k cột:

k−r

hu
Để tính tổng các số được điền vào phần tăng thêm, bao gồm phần ngang và phần dọc (giao nhau
T
ở một bảng k × k tại góc). Ta thấy phần nằm ngang có thể chia thành một số bảng vuông k × k
không chồng lên nhau và dư lại một hình chữ nhật kích thước k × (k − r) ở cuối. Tương tự, phần
t
nằm dọc có thể chia thành một số bảng vuông k × k không chồng lên nhau và dư lại một hình
Tấ

chữ nhật kích thước (k − r) × k.


Do đó, để tính tổng các số trên phần được thêm vào, trước hết, ta tính tổng các số trên các
hình chữ nhật không chồng lên nhau (theo giả thiết thì tổng đó bằng 0). Phần hình vuông kích
n

thước (k − r) × (k − r) có màu đỏ ở dưới là giao của 2 hình vuông cuối dãy ngang và đầu dãy
yễ

dọc, được tính 2 lần nên cần phải trừ ra. Cuối cùng, ta cộng các số trên bảng vuông ở góc có
kích thước r × r còn lại.
gu

Nói tóm lại, nếu đặt T là tổng các số trên bảng vuông k × k ở góc trên bên phải và S là tổng
các số trên phần chung là bảng (k − r) × (k − r) thì tổng tăng thêm là: T − S.
N

Khi đó, theo nhận xét 1 thì T − S ≤ t + s.


Nhận xét được chứng minh.
Trở lại bài toán,
Ta thấy trong phần màu xanh r × r ban đầu có tổng các số không vượt quá t nên tổng các
số của bảng sau a lần tăng kích thước (từ r × r lên n × n với n = ak + r) thì tổng sẽ không vượt
quá t + a(t + s).
Đến đây, ta sẽ chỉ ra một cách xây dựng bảng thỏa mãn.
Xét bảng ô vuông k × k ở góc trên bên phải của bảng n × n, ta điền như sau:

45
1. CỰC TRỊ TRONG TỔ HỢP

C A r

B D k−r

• Điền vào phần A là hình vuông r × r sao cho tổng các số trên đó là t (có thể điền t số là 1
và phần còn lại là 0).

• Điền vào phần B là hình vuông (k − r) × (k − r) sao cho tổng các số trên đó là −s (có thể
điền s số là −1 và phần còn lại là 0).

• Điền vào phần C,D tùy ý sao cho tổng của chúng bằng s − t. Rõ ràng tồn tại cách điền
như vậy do giới hạn giá trị của t,s.

hu
Đến đây, ta thực hiện điền số vào các bảng ô vuông kề với nó một cách tuần hoàn theo chu
kỳ k. Nghĩa là các số điền trên ô (a,b) sẽ giống số điền trên các ô

(a + k,b),(a,b + k),(a − k,b),(a,b − k).


T
t
Tấ

Dễ thấy rằng khi đó, tất cả các hình vuông kích thước k × k đều có tổng bằng 0 và hình vuông
r × r ở góc dưới bên trái giống với hình vuông r × r ở góc trên bên phải, tức là tổng bằng t.
Do đó, cách điền này cho ta bảng có tổng đúng bằng
n
yễ

t + a(t + s).
gu

Vậy giá trị lớn nhất cần tìm là t + a(t + s) với t,s,a được xác định như trên. 

Câu 1.39.
N

a) Gọi a1 ,a2 , . . . ,a20 là số viên kẹo của loại kẹo thứ 1,2, . . . ,20 với ai ≥ 2.
Với loại kẹo thứ i (1 ≤ i ≤ 20), ta đếm số bộ (A,B) mà hai học sinh A,B đều có loại kẹo
này. Số bộ cần đếm là C2ai .
X20 20
X
Khi đó, theo giả thiết, tổng số bộ chính là M hay M = C2ai trong đó ai = 2020.
i=1 i=1
Áp dụng bất đẳng thức Bu-nhi-a-cốp-ki ta có
20 20 20
X ai (ai − 1) X a2 i
X ai
M= = −
i=1
2 i=1
2 i=1
2
20
!2
X
ai 20
i=1
X ai 20202 2020
≥ − = − = 101000.
2 · 20 i=1
2 2 · 20 2

Dấu "=" xảy ra khi ai = 101,∀i = 1,2, . . . ,20.


Vậy giá trị nhỏ nhất của M là 101000.

46
1. CỰC TRỊ TRONG TỔ HỢP

19 19 19
X ai (ai − 1)
1X 2 1X
b) Như lý luận ở câu a, ta có M = ai −= ai .
i=1
2 2 i=1
2 i=1
X 19
Nên biểu thức M đạt giá trị nhỏ nhất khi và chỉ khi a2i đạt giá trị nhỏ nhất.
i=1
19
X
Ta sẽ chứng minh a2i đạt giá trị nhỏ nhất khi |ai − aj | ≤ 1 với mọi 1 ≤ i,j ≤ 19. (1)
i=1

Thật vậy. Xét bộ 4 số a, b, c, d mà a ≥ b + 2; c = a − 1; d = b + 1 thì ta có


cd = ab + a − b − 1 ≥ ab và (a + b)2 = (c + d)2 suy ra a2 + b2 ≥ c2 + d2 .
Mở rộng tính chất này cho nhiều số ta suy ra (1) được chứng minh.
Do đó M đạt giá trị nhỏ nhất khi có t số có giá trị là k và 19 − t số có giá trị là k + 1 với
0 ≤ t ≤ 19 và giá trị nhỏ nhất là
1 2
tk + (19 − t)(k + 1)2 − 2020 .

M=
2
Ta có tk + (19 − t)(k + 1) = 2020 ⇔ t = 19k − 2001.
2001 2020
Do 0 ≤ t ≤ 19 nên ≤k≤ .
19 19

hu
Từ đây ta có k = 106, t = 13.
1
Thay vào ta được giá trị nhỏ nhất của M là [13 · 1062 + 6 · 1072 − 2020] = 106371.
2

Câu 1.40. Ta chứng minh nmax = 66. Giả sử nmax > 66.
T 
t
Cách 1. Khi đó tồn tại đại biểu A100 quen 67 đại biểu A1 ,A2 , . . . ,A67 .
Tấ

Gọi X là tập hợp các đại biểu A68 ,A69 , . . . ,A99 thì |X| = 32.
Khi đó các đại biểu A1 ,A2 , . . . ,A67 chỉ có thể quen A100 và các đại biểu thuộc X (vì các đại biểu
Ai ,Aj với 1 ≤ i < j ≤ 67 không thể quen nhau).
n

Suy ra họ quen không quá 33 đại biểu.


yễ

Vì vậy những đại biểu quen 34, 35, . . . , 65, 66 đại biểu khác phải thuộc X. Điều này vô lý bởi
|X| = 32 < 66 − 34 + 1.
gu

Cách 2. Xét người A có 67 người quen. Xét người B có nhiều hơn hoặc bằng 34 người quen.
Nếu A quen B.
Khi đó trong 98 người còn lại có 66 người quen A và hiều hơn hoặc bằng 33 người quen B.
N

Do 66 + 33 = 99 > 98 nên A, B phải có người quen chung, mâu thuẫn.


Vậy A không quen B.
Vì có ít nhất 33 người có số người quen tương ứng là 34, 35, . . . , 66 nên có ít nhất 33 người không
quen A.
Điều này mâu thuẫn vì A quen với 67 người.
Vậy nmax ≥ 66 .
Ta xây dựng ví dụ thỏa mãn n = 66 như sau
A1 quen với B1 , B2 , . . . , B66 , A2 quen với B2 , B3 , . . . , B66 , . . ., A34 quen với B34 , B66 . Khi đó không
có 3 người đôi một quen nhau và Ak quen với 67 − k người (∀k = 1, . . . ,34) và Bk quen với k
người (∀k = 1, . . . ,34) và Bk quen 34 người với (∀k = 35, . . . ,66).
Rõ ràng trường hợp trên thỏa mãn các yêu cầu đề bài.

Câu 1.41. Viết các số từ 1 đến 20182 vào bảng ô vuông 2018 × 2018 như sau:

1 2 3 ··· 2017 2018


2019 2020 2021 ··· 4035 4036
··· ··· ··· ··· ··· ···
··· ··· ··· ··· ··· 20182

47
1. CỰC TRỊ TRONG TỔ HỢP

Khi đó ta thấy bộ 4 số thỏa mãn là đỉnh của một tứ giác đẹp phải là bốn ô vuông liên tiếp trên
một cột hoặc một hàng của bảng trên. Ta gọi bộ 4 số đó là một bộ "đẹp"
Vậy để tìm số bộ "đẹp" thì ta lát bảng ô vuông 2018 × 2018 trên bởi bảng ô vuông con 1 × 4 (1
hàng, 4 cột) hoặc 4 × 1 (4 hàng, 1 cột), gọi chung là các quân domino, sao cho không có hai quân
domino nào có ô vuông chung vì mỗi điểm thuộc tập A chỉ thuộc duy nhất một tứ giác nên mỗi
số chỉ có thể thuộc đúng một bộ đẹp.
Ta đếm số quân domino tối đa có thể dùng, hiển nhiên số quân domino tối đa sẽ không vượt quá
20182 : 4 = 10092 .
Tô đen các ô thuộc hàng chẵn hoặc cột chẵn của bảng ô vuông trên và tô đỏ các ô còn lại, khi
2018 2018
đó số các ô được tô đỏ là × = 10092 .
2 2
Để ý rằng mỗi một quân domino sẽ chứa đúng 0 hoặc 2 ô tô đỏ và số ô tô đỏ là lẻ nên ta không
thể lát kín bảng ô vuông bằng các quân domino trên, tức là số quân domino tối đa không thể là
10092 .
Ta chỉ ra một cách lát sử dụng 10092 − 1 quân domino gồm hai bước như sau:

• Bước 1: Lát hoàn toàn 2016 hàng đầu tiên của bảng chỉ với quân domino 4 × 1, cứ 4 hàng
liên tiếp thì ta lát bởi 2018 quân domino thì ta dùng 2018 × (2016 : 4) = 1009 × 1008 quân
domino.

hu
• 
Bước 2: Lát hai hàng cuối của bảng chỉ với quân domino 1 × 4, mỗi hàng lát liên tiếp bởi
2018
= 504 và trừ đi hai ô cuối hàng nên ta dùng tới 504 × 2 = 1008 quân domino.
4

Khi đó ta dùng 1008 × 1009 + 1008 = 1008 × 1010 = 10092 − 1 quân domino.
T
t
Vậy số tứ giác "đẹp" lớn nhất được tạo thành là 10092 − 1. 
Tấ

Câu 1.42. Nhận xét: Hai vận động viên bất kì tham gia chung không quá một môn thi đương
đương với hai môn thi bất kì chung nhau không quá một vận động viên.
n

Ta đưa về việc đếm bộ S có dạng (X,Y,Z) với hai môn X,Y có vận động viên Z tham gia chung.
yễ

2
• Đếm theo môn thi đấu, ta có S ≤ C12 .
gu

• Đếm theo vận động viên: ta gọi x1 ,x2 , . . . ,xn lần lượt là số môn mà vận động viên 1, 2,. . . ,n
tham gia.
Bằng cách đếm số lượt tham gia ta có
N

x1 + x2 + . . . + xn = 288.

Điều này dẫn đến


1
S = Cx21 + Cx22 + . . . + Cx2n = (x21 + x22 + . . . + x2n − 288).
2
Qua hai cách đếm trên ta suy ra

2 1
C12 ≥ S = (x21 + x22 + . . . + x2n − 288).
2
hay
x21 + x22 + . . . + x2n ≤ 420.

Áp dụng bất đẳng thức Cauchy – Schwarz, ta có

(x1 + x2 + . . . + xn )2 2882
x21 + x22 + ... + x2n ≥ = .
n n

48
1. CỰC TRỊ TRONG TỔ HỢP

Suy ra
2882
420 ≥ hay n ≥ 198.
n
 
288
Để ý rằng với n ≥ 198 thì ước lượng được = 1, khi đó ta dự đoán cực trị xảy ra khi
n
các số xi nhận giá trị là 1 hoặc 2. Do vậy, với mỗi i ∈ {1,2, . . . ,n} ta đánh giá

(xi − 1)(xi − 2) ≥ 0 ⇔ x2i ≥ 3xi − 2, ∀i = 1,n.

Đến đây, ta suy ra

420 ≥ x21 + x22 + . . . + x2n ≥ (3x1 − 2) + (3x2 − 2) + . . . + (3xn − 2)


= 3(x1 + x2 + . . . + xn ) − 2n = 3.288 − 2n.

và thu được n ≥ 222.


Vì bất đẳng thức xảy ra khi có 156 số xi bằng 1 và 66 số xi bằng 2 nên giá trị nhỏ nhất
cần tìm là 222.

hu
Câu 1.43. Tổng số quả mà 100 chú gấu hái được là

T
1 + 2 + · · · + 100 = 5050 (quả).

Do khi các chú gấu có số quả bằng nhau, mỗi chú gấu đều có ít nhất một quả nên số quả sói có
t
thể ăn không vượt quá
Tấ

5050 − 100 = 4950 (quả).

Ta sẽ chứng minh sói có cách “ăn chia” để số quả được ăn bằng 4950.
n

Ta có nhận xét sau:


yễ

Nhận xét. Giả sử có k chú gấu (k ≥ 2), được đặt tên theo thứ tự từ bé đến lớn bởi “1”, “2”,
. . . , “k” tương ứng có số quả là 1,2, . . . ,k. Khi đó, bằng cách thực hiện quy trình “ăn chia” dưới
đây, sói có thể ăn k − 1 quả và làm cho số quả của các chú gấu “1”, “2”, . . ., “k − 1”, “k” tương
gu

ứng là 1,1,2, . . . ,k − 2,k − 1:


Quy trình “ăn chia” của sói: Với mỗi t = 1,2, . . . ,k − 1, ở bước thứ t, sói gộp số quả của hai chú
N

gấu “k − t + 1” và “k − t” với nhau, rồi “ăn chia” theo nguyên tắc đã nêu trong đề bài.
Chứng minh. Bằng quy nạp theo t, 1 ≤ t ≤ k − 1, dễ dàng chứng minh được khẳng định sau:
“Ở mỗi bước của quy trình, sói được ăn 1 quả và sau bước thứ t, số quả của các chú gấu “1”, . . .,
“k − t”, “k − t + 1”, . . ., “k” tương ứng là 1, . . . ,k − t,k − t, . . . , k − 1 ”.
Từ khẳng định trên, hiển nhiên suy ra kết luận của Nhận xét.
Đặt tên 100 chú gấu, theo thứ tự từ bé đến lớn, bởi “1”, “2”, . . ., “100”.
Từ nhận xét dễ dàng suy ra sói có thể thực hiện việc “ăn chia” theo cách sau:

• Lần 1: Sói thực hiện quy trình “ăn chia” nêu trên đối với 100 chú gấu (k = 100);

• Lần 2: Sói thực hiện quy trình cho 99 chú gấu, từ chú gấu “2” đến chú gấu “100”(k = 99);

• ···

• Lần thứ m: Sói thực hiện quy trình cho 100 − m + 1 chú gấu, từ chú gấu “m” đến chú gấu
“100” (k = 100 − m + 1);

• ···

• Lần thứ 99: Sói thực hiện quy trình cho 2 chú gấu, chú gấu “99” và chú gấu “100” k = 2).

49
1. CỰC TRỊ TRONG TỔ HỢP

Bằng quy nạp theo m, 1 ≤ m ≤ 99, dễ dàng chứng minh được khẳng định sau:
“Với cách thực hiện quy trình như trên, sau lần thực hiện thứ m, sói sẽ ăn được số quả là”

99 + 98 + · · · + (100 − m) + (100 − m + 1) (quả)

và số quả của 100 chú gấu “1”, “2”, . . ., “100” tương ứng là

1,1, . . . ,1,1,2,3, . . . ,100 − m.

Từ khẳng định vừa nêu suy ra, sau lần thực hiện quy trình thứ 99, số quả của 100 chú gấu cùng
bằng 1 và số quả sói ăn được là:

99 + 98 + · · · + 2 + 1 = 4950 (quả).

Vậy, tóm lại, bằng cách thực hiện đúng quy trình “ăn chia” sói có thể ăn được tối đa 4950 quả. 

Câu 1.44. Với n là số nguyên dương và p là số nguyên tố, ta kí hiệu vp (n) là số lớn nhất trong
các số tự nhiên k mà pk |n. Dễ thấy vp (mn) = vp (m) + vp (n), với mọi số nguyên dương m, n và
với mọi số nguyên tố p. Giả sử x1 < x2 < · · · < xn là các số nguyên dương vừa thuộc S vừa là
các số hạng của một cấp số nhân tăng, có công bội q > 1. Vì 1,2,4,8,16 ∈ S và 1,2,4,8,16 cũng
là các số hạng của một cấp số nhân tăng có công bội lớn hơn 1 nên có thể giả sử n ≥ 5. Vì

hu
x1 < x2 < · · · < xn là các số hạng của một cấp số nhân tăng với công bội q nên tồn tại các số
nguyên dương a1 , a2 , . . . ,an sao cho

với mọi i = 1,n − 1 hay


xi+1
xi+1 = xi · q ai ,
T
= q ai với mọi i = 1,n − 1. Suy ra, với mỗi i = 1,2, . . . ,n − 2, ta có
t
xi
Tấ

 a  a
xi+1 i+1 ai ·ai+1 xi+2 i
=q = .
xi xi+1
n

a a +ai+1
Do đó, xi i+1 · xai+2
i i
= xi+1 với mỗi i = 1,2, . . . ,n − 2. Vì vậy, với p là số nguyên tố tùy ý, ta có
yễ

(ai + ai+1 ) vp (xi+1 ) = ai+1 vp (xi ) + ai vp (xi+2 )


gu

với mỗi i = 1,2, . . . ,n − 2. Suy ra, với p là số nguyên tố tùy ý, ta có


N

vp (xi ) < vp (xi+1 ) ⇔ vp (xi+1 ) < vp (xi+2 )

với mọi i = 1,n − 2. Dễ thấy, tồn tại số nguyên tố p sao cho vp (x1 ) < vp (x2 ) (vì nếu ngược lại,
vp (x1 ) > vp (x2 ) với mọi số nguyên tố thì x2 |x1 , trái với giả thiết x1 < x2 ). Từ đó, suy ra tồn tại
số nguyên tố p sao cho
vp (x1 ) < vp (x2 ) < · · · < vp (xn ) .
Do đó, vp (xn ) ≥ n − 1. Suy ra pn−1 |xn . Mà xn ≤ 2017 nên pn−1 ≤ 2017. Vì thế, 2n−1 ≤ 2017.
Suy ra, n ≤ 11. Dễ thấy, 1,2,22 , . . . ,210 ∈ S và các số đó lập thành một cấp số nhân tăng, có
công bội lớn hơn 1. Vậy, có tối đa 11 phần tử thuộc S là các số hạng của một cấp số nhân tăng,
có công bội lớn hơn 1. 

Câu 1.45. Dễ thấy, với a là số nguyên dương, ta có:

• Nếu a là số chẵn (tức a chia hết cho 4 hoặc chia 4 dư 2) thì a3 chia hết cho 4; (1)

• Nếu a là số lẻ (tức a chia 4 dư 1 hoặc chia 4 dư 3) thì a3 chia 4 dư 1 (nếu a chia 4 dư 1)


hoặc a3 chia 4 dư 3 (nếu a chia 4 dư 3). (2)

50
1. CỰC TRỊ TRONG TỔ HỢP

Xét một cách chọn bất kì một số số thỏa mãn yêu cầu đề bài, từ 2017 số nguyên dương đầu tiên.
Giả sử số được chọn là n (n ≥ 2) và trong đó có m số chẵn, k số lẻ. Do tổng lập phương của 2
số bất kì trong n số được chọn không chia hết cho 4, nên từ (1) và (2) suy ra, với a, b là hai số
nguyên dương tùy ý trong n số đó, ta sẽ có hoặc a, b khác tính chẵn lẻ, hoặc a, b cùng lẻ và đồng
thời có cùng số dư khi chia cho 4. Từ đó, do trong 2017 số nguyên dương đầu tiên có 505 số chia
4 dư 1 và 504 số chia 4 dư 3, nên suy ra ta có:

• m ≤ 1 (vì nếu m ≥ 2 thì trong n số được chọn tồn tại 2 số cùng chẵn);

• k ≤ 505 (vì nếu k ≥ 506 thì trong n số được chọn tồn tại 2 số cùng lẻ và đồng thời có số
dư khác nhau khi chia cho 4).

Như vậy, n = m + k ≤ 1 + 505 = 506. Dễ thấy, từ 2017 số nguyên dương đầu tiên, chọn ra 506
số gồm 505 số chia 4 dư 1 và một số chẵn (tùy ý) ta sẽ có 506 số thỏa mãn yêu cầu đề bài. Vậy,
có thể chọn ra tối đa 506 số từ 2017 số nguyên dương đầu tiên sao cho tổng lập phương của hai
số bất kì trong các số được chọn là một số không chia hết cho 4.
Nhận xét. Kết quả quen thuộc sau đây đã được sử dụng để tìm số các số chia 4 dư 1 và số các
số chia 4 dư 3, trong số 2017 số nguyên dương đầu tiên: “Cho m, k, n là các số nguyên dương,
với 1 ≤ m ≤ k ≤ n. Kí hiệu s là số các số nguyên dương chia k dư m, trong n số nguyên dương

hu
đầu tiên; ta có:  
n−m+k
s= .
k
([x] kí hiệu số nguyên lớn nhất không vượt quá x).” T
t

Tấ

Câu 1.46. Giả sử các điểm được tô bởi hai màu, đen và trắng. Xét 8 điểm A1 , A2 ,...,A8 , được
tô màu như sau: A1 , A2 ,A5 ,A − 6 được tô màu trắng; A3 , A4 ,A7 ,A8 được tô bởi màu đen. Dễ
thấy, không có ba điểm Ai ,Aj ,A2j−i (1 ≤ i < 2j − i ≤ 8) nào được tô cùng màu. Do đó n ≥ 9.
n

Ta sẽ chứng minh n = 9 là số có tính chất như đề bài yêu cầu. Thật vậy, giả sử ngược lại,
yễ

tồn tại cách tô 9 điểm A1 , A2 , . . . ,A9 bởi hai màu, đen và trắng sao cho không có ba điểm
Ai ,Aj ,A2j−i (1 ≤ i < 2j − i ≤ 9) nào được tô cùng màu.
gu

Trường hợp 1: Tồn tại i ∈ {3; 4; 5}, sao cho các điểm Ai và Ai+2 được tô cùng màu, giả sử là màu
trắng. Khi đó, các điểm Ai−2 ,Ai+1 , Ai+4 phải được tô màu đen (chú ý i − 2 ≥ 1
N

và i + 4 ≤ 9), vô lý.

Trường hợp 2: Với mọi i ∈ {3; 4; 5}, các điểm Ai và Ai+2 được tô khác màu. Không mất tính
tổng quát, giả sử A5 được tô màu trắng. Khi đó, A3 và A7 được tô màu đen. Nhờ
tính đối xứng, ta có thể giả sử rằng A4 được tô màu trắng và A6 được tô màu
đen. Khi đó A8 được tô màu trắng (do A6 , A7 và A8 không được tô cùng màu);
suy ra, A2 được tô đen (do A2 , A5 và A8 không được tô cùng màu) và A9 được
tô trắng (do A7 ,A8 và A9 không được tô cùng màu). Do đó, A1 phải vừa được tô
màu trắng (do A1 ,A2 và A4 không được tô cùng màu),vừa được tô đen (do A1 , A5
và A9 không được tô cùng màu), là điều vô lý.

Những điều vô lí nhận nhận được ở trên cho ta điều muốn chứng minh.
Vậy tóm lại, n = 9 là các số nguyên nhỏ nhất có tính chất như đề bài yêu cầu.
Nhận xét.

1 Những nhược điểm về suy luận lôgic thường gặp khi giải bài này:

- Do không ý thức được rằng nếu số nguyên n ≥ 4 không có tính chất như đề bài yêu cầu thì
số nguyên n − 1 cũng không có tính chất đó, học sinh thường cồng kềnh hóa lời giải bằng
việc xét lần lượt các số nguyên thuộc tập hợp {3; 4; 5; 6; 7; 8}.

51
1. CỰC TRỊ TRONG TỔ HỢP

- Nhiều học sinh do chưa hiểu rõ khái niệm giá trị lớn nhất, giá trị nhỏ nhất nên đã giải bài
toán theo lượt đồ sau:
+ Chứng minh số n thuộc tập hợp {3; 4; 5; 6; 7; 8} không có tính chất như đề bài yêu cầu;
+ Chứng minh n = 9 là số có tính chất như đề bài yêu cầu;
+ Chứng minh tất cả các số nguyên n > 9 đều có tính chất như đề bài yêu cầu;
+ Từ chứng minh ở các bước trên, suy ra n = 9 là số nguyên nhỏ nhất cần tìm.

2 Có thể phát biểu bài toán này dưới dạng:


“Tìm số nguyên n ≥ 3 nhỏ nhất có tính chất: Với mọi cách phân chia tập hợp gồm n số nguyên
dương đầu tiên thành hai tập con khác rỗng, luôn tồn tại một dãy số cách đều gồm 3 số hạng
mà tất cả các số hạng của nó đều thuộc cùng một tập tập con.”
Dạng phát biểu trên gợi ý cho ta đặt ra và tìm cách giải quyết bài toán khái quát sau:
Bài toán khái quát. Cho số nguyên k ≥ 3. Tìm số nguyên n ≥ k nhỏ nhất có tính chất: Với
mọi cách phân chia tập hợp gồm n số nguyên dương đầu tiên thành hai tập con khác rỗng,
luôn tồn tại một dãy số cách đều gồm k số hạng mà tất cả các số hạng của nó đều thuộc cùng
một tập tập con.
Lưu ý rằng, hướng khái quát trên không phải là duy nhất.

hu


Câu 1.47. Chia tập hợp {1, 2, 3, . . . , 4013, 4014} thành 2007 phần P1 ∪ P2 ∪ . . . ∪ P2007 (mỗi

T
tập hợp chứa ít nhất một phần tử của {1, 2, 3, . . . , 4013, 4014}) thỏa mãn tập hợp Pa chứa tất
cả các số nguyên dương có dạng 2n (2a − 1), trong đó n là một số không âm. Khi đó, tập hợp
t
con A của {1, 2, 3, . . . , 4013, 4014} không thể chứa hai phần tử cùng thuộc một trong 2007 tập
Tấ

hợp đó vì nếu không thì rõ ràng có một số sẽ chia hết cho số còn lại, mâu thuẫn. Mặt khác, A
lại có đúng 2007 phần tử nên A chứa đúng 1 phần tử của mỗi tập Pa nói trên.
Gọi αi là một phần tử của A với αi ∈ Pi , i = 1, 2007. Xét các phần tử α1 , α2 , α5 , . . . α1094 lần
lượt có các dạng 2n , 3.2n , 32 .2n , . . . , , 37 .2n ; rõ ràng mỗi phần tử đó chỉ có hai ước nguyên tố là
n

2 và 3. Ta cũng thấy rằng lũy thừa lớn nhất của 2 trong α1 phải lớn hơn lũy thừa lớn nhất của 2
yễ

.
trong α2 vì nếu ngược lại thì α2 ..α1 , mâu thuẫn.
Hoàn toàn tương tự với các phần tử khác trong dãy α1 , α2 , α5 , . . . α1094 , tức là nếu i < j thì
gu

lũy thừa của 2 trong αi phải lớn hơn lũy thừa của 2 trong αj . Do đó, lũy thừa của 2 trong dãy
α1 , α2 , α5 , . . . α1094 là một dãy giảm thực sự. Do có 8 phần tử trong dãy trên (tương ứng với lũy
N

thừa của 3 thay đổi từ 0 đến 7) nên giá trị của α1 ít nhất là 27 = 128. Hơn nữa, các phần tử của
dãy có giá trị của lũy thừa 3 tăng dần (nếu ngược lại, giá trị của lũy thừa 3 giảm mà lũy thừa 2
cũng giảm nên có một số chia hết cho số khác, mâu thuẫn).
Suy ra: α1 chính là giá trị nhỏ nhất trong dãy trên.
Hoàn toàn tương tự với các số nguyên tố khác, ta cũng xét số có dạng αi = 2ni .3ni · · · pknk , trong
đó 2, 3, . . . , pk là các ước nguyên tố không vượt quá 4014. Từ đó suy ra α1 cũng chính là phần
tử nhỏ nhất trong các phần tử của tập A. Do đó phần tử nhỏ nhất mA của tập A chính là α1 ,
như chứng minh ở trên α1 ≥ 128 hay mA ≥ 128.
Ta sẽ chứng minh rằng 128 chính là giá trị nhỏ nhất của mA bằng cách chỉ ra một tập hợp con
A của {1, 2, 3, . . ., 4013, 4014} thỏa mãn điều kiện đề bài.
Xét tập hợp: A = αi = 2f (i) (2i − 1)|1338 ≤ 3f (i) .(2i − 1) ≤ 4014 .
Rõ ràng tập hợp này có đúng 2007 phần tử thuộc {1, 2, 3, . . . , 4013, 4014}.
Ta sẽ chứng minh rằng αx không chia hết cho αy , với x > y (tức là tập hợp này thỏa mãn đề
bài).
.
Thật vậy, giả sử ngược lại tồn tại x, y thỏa mãn αx ..αy , khi đó:
.
2f (x) (2x − 1)..2f (y) (2y − 1),

52
1. CỰC TRỊ TRONG TỔ HỢP

.
tức là f (x) ≥ f (y) và (2x − 1)..(2y − 1).
.
Từ cách xác định các giá trị u, v ta có: 4(2x − 1)..(2y − 1) ⇒ 2x − 1 ≥ 3(2y − 1), đồng thời
43f (y)+1 (2y − 1) ≥ 4014 > 3f (x) (2x − 1) ≥ 3f (x)+1 (2y − 1) ⇒ f (y) > f (x).
Mâu thuẫn này chứng tỏ tất cả các phần tử của A đều thỏa mãn với mọi a, b ∈ A thì a không
chia hết cho b.
Vậy 128 chính là giá trị nhỏ nhất của mA cần tìm. 
Câu 1.48. Trước hết ta thấy rằng: Nếu S là tập khuyết trong T thì tập S 0 = {n − x|x ∈ S}
cũng là một tập khuyết trong T .
Thật vậy: Giả sử ngược lại S 0 không phải là tập khuyết, khi đó tồn tại hai số nguyên dương
s01 , s02 ∈ S 0 sao cho
|s0 1 − s0 2 | = c
n
với c là một số nguyên dương nào đó không vượt quá . Khi đó xét tương ứng hai phần tử
2
s1 = n − s01 ,s2 = n − s02 thì rõ ràng s1 ,s2 ∈ S và
|s1 − s2 | = |(n − s0 1 ) − (n − s0 2 )| = |s0 1 − s0 2 | = c,
tức là tồn tại hai phần tử s1 , s2 ∈ S và

hu
n
|s1 − s2 | = c ≤
2

T
trong khi S là tập khuyết. Mâu thuẫn này suy ra nhận xét trên được chứng minh.
Hơn nữa, do |S| = |S 0 | nên khi S có số các phần tử là lớn nhất thì tương ứng cũng có tập S 0 có
t
số phần tử lớn nhất bằng với S.
n
Tấ

Từ đó, ta thấy có thể xét các tập khuyết S có số các số phần tử không vượt quá không ít hơn số
n n no n2 no
các số phần tử lớn hơn . Xét hai tập hợp sau: A = x | x ∈ S, x ≤ , B = x | x ∈ S, x >
2 2 2
thì A ∩ B = ∅,A ∪ B = S và theo cách xác định S như trên thì |A| ≥ |B|.
n

n
Khi đó với c là một số nguyên dương nào đó không vượt quá , ta xét tập hợp: C = {x+c|x ∈ A}.
yễ

2
Ta có: |A| = |C|. Do A ⊂ S nên A cũng là một tập khuyết và khi đó rõ ràng A∩C = ∅,B ∩C = ∅
(vì nếu ngược lại thì tồn tại hai phần tử thuộc S mà hiệu của chúng là c, mâu thuẫn).
gu

Suy ra tất cả các phần tử thuộc tập A hoặc B hoặc C đều là một số nguyên dương không vượt
quá n, tức là
N

(A ∪ B ∪ C) ⊂ T ⇒ |A| + |B| + |C| ≤ |T | = n.


Kết hợp các điều này lại, ta được: 2 |A| + |B| ≤ n. Do đó:
4 |A| + 2 |B| 2n
|A| + |B| ≤ ≤ .
3 3
Hơn nữa: A ∩ B = ∅, A ∪ B = S và |S| là số nguyên nên
 
2n
|S| = |A| + |B| ≤ .
3
 
2n
Do đó số phần tử của tập khuyết S trong T luôn không vượt quá .
  3
2n
Ta sẽ chỉ ra một tập khuyết thỏa mãn đề bài có đúng phần tử.
3
Thật vậy, xét hai tập hợp A, B như sau:
  
n+1 n hni hni o
A = 1,2,3,..., , B = n− + 1, n − + 2,...,n
3 3 3
 
n+1 n
và S = A ∪ B. Chọn c = ≤ . Ta thấy:
3 2

53
1. CỰC TRỊ TRONG TỔ HỢP

  
n+1 n+1
• Hiệu hai phần tử bất kì trong A không vượt quá −1< .
3 3

• Hiệu hai phần tử bất kì trong B không vượt quá


hni hni  
n+1
n − (n − + 1) = −1< .
3 3 3

• Hiệu một phần tử bất kì thuộc B với một phần tử bất kì thuộc A không nhỏ hơn:
hni    
n+1 n n+1 n+2 n+1
(n − + 1) − ≥n− − +1= > .
3 3 3 3 3 3
 
n+1 n
Khi đó, rõ ràng S = A ∪ B là một tập khuyết trong T ứng với giá trị c = ≤ .
  3 2
2n
Ta sẽ chứng minh rằng |S| = . Từ cách xác định A, B, ta có:
3
 
n+1 hni
|A| = ,B = .
3 3

hu
Ta cần có:   h i  
n+1 n 2n

Xét các trường hợp:


3
+
3
=
3
T
,∀n ∈ N. (∗)
t
Tấ

• Nếu n chia hết cho 3, tức là n có dạng 3m,m ∈ N. Suy ra:


  h i      
n+1 n 3m + 1 3m 6m 2n
+ = + = m + m = 2m = = .
3 3 3 3 3 3
n
yễ

• Nếu n chia 3 dư 1, tức là n có dạng 3m + 1,m ∈ N. Suy ra:


  h i        
n+1 n 3m + 2 3m + 1 6m 6m + 2 2n
gu

+ = + = m + m = 2m = = = .
3 3 3 3 3 3 3
N

• Nếu n chia 3 dư 2, tức là n có dạng 3m + 2,m ∈ N. Suy ra:


  h i        
n+1 n 3m + 3 3m + 2 6m + 3 6m + 4 2n
+ = + =m+1+m= = = .
3 3 3 3 3 3 3


2n
Từ đó suy ra (*) được chứng minh hay tập hợp S đã cho là tập khuyết có .
  3
2n
Vậy giá trị lớn nhất của số phần tử của tập khuyết S trong T là . 
3

Câu 1.49.
a) Với m = 4, ta sẽ chứng minh rằng số máy bơm nước nhỏ nhất thỏa mãn yêu cầu đề bài là
n.
Điều kiện đủ là hiển nhiên với cách đặt ở mỗi cột 1 máy bơm ở hàng thứ hai như sau:

·········
X X X ········· X X X
·········
·········

54
1. CỰC TRỊ TRONG TỔ HỢP

Chú ý là một máy bơm chỉ tưới được tối đa 6 ô nên điều kiện cần rõ ràng đúng với n = 1
và n = 2.
Ta chứng minh điều kiện cần bằng phản chứng. Giả sử tồn tại n sao cho cánh đồng kích
thước 4 × n có thể tưới được bằng ít hơn n máy bơm nước. Gọi n0 là số nguyên dương n
nhỏ nhất như vậy. Theo chú ý ở trên ta phải có n0 ≥ 3.
Xét cánh đồng kích thước 4 × n0 . Theo định nghĩa của n0 , tồn tại một cách xếp k < n0
máy bơm để tưới hết cánh đồng. Vì số máy bơm nhỏ hơn số cột nên phải tồn tại ít nhất
một cột không chứa máy bơm (ta gọi là cột trống).
• Ta thấy cột trống không thể là cột ở biên vì nếu cột trống là cột biên, chẳng hạn là
cột thứ nhất thì để tưới được các ô ở cột trống, cột thứ hai phải chứa 4 máy bơm. Khi
đó, bằng cách thêm một máy bơm vào cột 3 hàng 2 (nếu ô này chưa có máy bơm), ta
thấy n0 − 2 cột còn lại (bỏ đi cột 1 và 2) sẽ được tưới bởi k − 4 + 1 = k − 3 < n0 − 2
máy bơm, mâu thuẫn với cách chọn n0 .
• Vì cột trống không nằm ở biên, ta xét cột trống đầu tiên từ bên trái sang. Ta giả sử
cột này là cột j. Để tưới được các ô ở cột trống này, tổng cộng ở hai cột hai bên cột
trống này phải có ít nhất 4 máy bơm. (*)
Xét các trường hợp sau:

hu
– Cột j − 1 chứa ít nhất 2 máy bơm. Khi đó do các cột từ 1 đến j − 2 đều trống nên
j cột đầu chứa ít nhất j máy bơm. Suy ra n0 − j cột sau chứa nhiều nhất k − j
máy bơm. Vì được ngăn cách bởi 1 cột trống nên rõ ràng các máy bơm này bơm

T
được cho tất cả các ô của cách đồng kích thước 4 × (n0 − j). Vì k − j < n0 − j
nên điều này mâu thuẫn với cách chọn n0 .
t
– Cột j − 1 chỉ chứa 1 máy bơm, khi đó, do (*), cột j + 1 phải chứa ít nhất 3 máy
Tấ

bơm. Khi đó, do j + 1 cột đầu chứa ít nhất j − 1 + 0 + 3 = j + 2 máy bơm


nên n0 > k ≥ j + 2, tức là bên cạnh cột j + 1 còn ít nhất 2 cột nữa. Bây giờ,
bằng cách thêm vào cột 2 hàng j + 2 một máy bơm nếu cần, ta thấy cánh đồng
n

gồm n0 − (j + 1) cột còn lại sau khi bỏ đi j + 1 cột đầu có thể được tưới bởi
k − (j + 2) + 1 = k − j − 1 máy bơm. Vì k − j − 1 < n0 − (j + 1) nên ta nhận
yễ

được mâu thuẫn với cách chọn n0 .


gu

Như vậy điều kiện cần được chứng minh. Ta có kết luận: Với cánh đồng 4 × n cần ít nhất
n máy bơm để tưới nước thỏa mãn yêu cầu bài toán.
N

 
n−1
b) Ta sẽ chứng minh rằng số máy bơm ít nhất để tưới được cánh đồng 3 × n là n − .
4
Trước hết ta chứng minh điều kiện đủ. Với n < 5 điều kiện đủ là hiển nhiên, ta xếp mỗi
cột 1 máy bơm là được.
Với n = 5, ta có cách xếp sau:

X X
X
X

Từ đây dễ dàng chỉ ra cách đặt máy bơm cho n bất kỳ. Chẳng hạn với n = 20, ta đặt 16
máy bơm như sau.

X X X X X X
X X X X X
X X X X X

Bây giờ ta chứng minh điều kiện cần. Chú ý là một máy bơm nước chỉ có thể tưới được tối
đa 5 ô nên với n = 1, 2, ta thấy rằng cần phải có ít nhất n máy bơm nước mới có thể tưới

55
1. CỰC TRỊ TRONG TỔ HỢP

 đồng 3 × n.
được tất cả các ô của cánh
n−1
Đặt f (n) = n − với n = 1,2, . . .
4
Giả sử tồn tại số nguyên dương n sao cho cánh đồng kích thước 3 × n có thể được tưới bởi
k < f (n) máy bơm. Gọi n0 là số nhỏ nhất như vậy. Theo chú ý trên ta có n0 ≥ 3.
Do f (n0 ) ≥ n0 nên ta có k < n0 , như vậy có ít nhất một cột trống.
Lý luận tương tự như ở phần 1, ta thấy cột trống không thể ở biên. Xét cột trống đầu tiên
từ bên trái sang. Giả sử đó là cột j. Khi đó, để tưới các ô của cột j, hai cột kề bên cột j
phải chứa ít nhất 3 máy bơm nước.
Xét các trường hợp sau:

• Cột j − 1 chứa ít nhất 2 máy bơm nước. Khi đó j cột đầu chứa ít nhất j máy bơm
nước (do các cột từ 1 đến j − 2 chứa ít nhất 1, cột j − 1 chứa ít nhất 2). Suy ra n0 − j
cột còn lại chứa không quá k − j máy bơm nước và các máy bơm này tưới hết các ô
của cánh đồng kích thước 3 × (n0 − j). Ta có
   
n0 − 1 n0 − j − 1
k − j < f (n0 ) − j = n0 − j − ≤ n0 − j − = f (n0 − j)
4 4

hu
nên từ đây ta suy ra điều mâu thuẫn với cách chọn n0 .
• Cột j − 1 chỉ chứa 1 máy bơm, khi đó, do đó cột j + 1 phải chứa ít nhất 2 máy bơm.

T
Như thế j + 1 cột đầu chứa ít nhất j + 1 máy bơm. Suy ra n0 − (j + 1) cột còn lại
chứa không quá k − (j + 1) máy bơm nước. Ta xét hai trường hợp sau
t
– Trường hợp 1: Cột j + 2 là cột trống. Khi đó n0 − (j + 2) cột còn lại sau khi bỏ
Tấ

j + 2 cột đầu được tưới đủ bởi nhiều nhất k − (j + 1) máy bơm nước. Ta có
 
n0 − 1
k − (j + 1) < f (n0 ) − (j + 1) = n0 − (j + 1) −
4
n

 
n0 − 4 − 1
yễ

= n0 − (j + 2) −
4
 
n0 − (j + 2) − 1
gu

≤ n0 − (j + 2) − = f (n0 − (j + 2))
4
N

(do j ≥ 2 nên j + 2 ≥ 4).


Điều này mâu thuẫn với cách chọn n0 .
– Cột j + 2 có ít nhất một máy bơm. Khi đó n0 − (j + 2) cột còn lại sau khi bỏ j + 2
cột n0 được tưới đủ các ô ở các cột này (có n0 − (j + 1) cột). Theo tính toán ở
trên, số máy bơm ở các cột này không quá k − (j + 1). Ta lại có đánh giá

k − (j + 1) < f (n0 ) − (j + 1)
 
n0 − 1
= n0 − (j + 1) −
4
 
n0 − (j + 1) − 1
≤ n0 − (j + 1) −
4
= f (n0 − (j + 1))

mâu thuẫn với cách chọn n0

Bài toán được giải quyết hoàn toàn.

56

You might also like